Top Banner
BYJU’S UPSC Monthly Magazine March 2022 Answer Key Q1. Which of the given statements best describes “NISHTHA”? a. An integrated scheme for school education focused on improving the quality of education at all levels. b. A teachers’ training programme launched to improve Learning Outcomes at the Elementary level. c. A set of guidelines issued for the Planning and implementation of Digital Education in India. d. A National Teacher Platform built to host Open Educational Resources (OER) and tools for teachers. Answer: b Explanation: NISHTHA is a capacity building programme for “Improving Quality of School Education through Integrated Teacher Training”. It aims to build competencies among all the teachers and school principals at the elementary stage. The basic objective of this massive training programme is to motivate and equip teachers to encourage and foster critical thinking in students. Hence B is the correct option. Q2. Which of the given statements is/are correct? 1. It is the only one of the six principal organs of the UN that is not located in New York City. 2. All members of the UN are automatically parties to the ICJ statute and this automatically gives the ICJ jurisdiction over disputes involving them. 3. It has 15 judges who are elected to nine-year terms by the UN General Assembly and Security Council. 4. India has never been a party to a case at the ICJ. Options: a. 1 and 4 only b. 2 and 3 only c. 1 and 3 only d. 1, 2 and 3 only Answer: c Explanation: ICJ is the principal judicial organ of the United Nations (UN). Established in June 1945 by the Charter of the United Nations and began work in April 1946. All members of the UN are automatically parties to the ICJ statute. However, the ICJ does not have jurisdiction over disputes involving them. The ICJ gets jurisdiction only if both parties consent to it. Hence, statement 2 is incorrect. The ICJ has 15 judges who are elected to nine-year terms by the UN General Assembly and Security Council, which vote simultaneously but separately. To be elected, a candidate must receive a majority of the votes in both bodies. Hence Statement 3 is correct. Elections are held at the UNHQ in New York during the annual UNGA meeting.
74

BYJU'S UPSC Monthly Magazine March 2022 Answer Key

Apr 30, 2023

Download

Documents

Khang Minh
Welcome message from author
This document is posted to help you gain knowledge. Please leave a comment to let me know what you think about it! Share it to your friends and learn new things together.
Transcript
Page 1: BYJU'S UPSC Monthly Magazine March 2022 Answer Key

BYJU’S UPSC Monthly Magazine March 2022

Answer Key

Q1. Which of the given statements best describes “NISHTHA”?

a. An integrated scheme for school education focused on improving the quality of education at all levels.

b. A teachers’ training programme launched to improve Learning Outcomes at the Elementary level.

c. A set of guidelines issued for the Planning and implementation of Digital Education in India. d. A National Teacher Platform built to host Open Educational Resources (OER) and tools for teachers.

Answer: b

Explanation:

NISHTHA is a capacity building programme for “Improving Quality of School Education through Integrated Teacher Training”.

It aims to build competencies among all the teachers and school principals at the elementary stage.

The basic objective of this massive training programme is to motivate and equip teachers to encourage

and foster critical thinking in students.

Hence B is the correct option.

Q2. Which of the given statements is/are correct?

1. It is the only one of the six principal organs of the UN that is not located in New York City.

2. All members of the UN are automatically parties to the ICJ statute and this automatically gives the ICJ jurisdiction over disputes involving them.

3. It has 15 judges who are elected to nine-year terms by the UN General Assembly and Security Council.

4. India has never been a party to a case at the ICJ.

Options:

a. 1 and 4 only b. 2 and 3 only

c. 1 and 3 only

d. 1, 2 and 3 only

Answer: c

Explanation:

ICJ is the principal judicial organ of the United Nations (UN).

Established in June 1945 by the Charter of the United Nations and began work in April 1946.

All members of the UN are automatically parties to the ICJ statute. However, the ICJ does not have

jurisdiction over disputes involving them. The ICJ gets jurisdiction only if both parties consent to it.

Hence, statement 2 is incorrect.

The ICJ has 15 judges who are elected to nine-year terms by the UN General Assembly and Security Council, which vote simultaneously but separately. To be elected, a candidate must receive a majority of

the votes in both bodies. Hence Statement 3 is correct.

Elections are held at the UNHQ in New York during the annual UNGA meeting.

Page 2: BYJU'S UPSC Monthly Magazine March 2022 Answer Key

A third of the court is elected every three years. In November 2017, India’s Dalveer Bhandari was re-

elected to the International Court of Justice (ICJ). Hence Statement 4 is not correct.

Hence C is the correct option.

Q3. Consider the following statements with respect to the ‘Donbas region’ recently in the news:

1. The region includes the states of Donetsk and Luhansk. 2. It is an industrial area and is famous for its coal reserves.

3. The region shares its borders with Russia and Belarus.

Which of the given statement/s is/are INCORRECT?

a. 1 only b. 2 and 3 only

c. 3 only

d. 1, 2 and 3

Answer: c

Explanation:

Donetsk and Luhansk are two states located in eastern Ukraine, which share a border with Russia. Hence

Statement 1 is correct.

This entire region, which includes Donetsk, Luhansk and their respective separatist territories, is generally

referred to as the ‘Donbas’ region.

It is an industrial area and is famous for its coal reserves. Hence Statement 2 is correct.

Donbas, located on the Russian border adjoining Russia, has a majority Russian-speaking population. It doesn’t share borders with Belarus. Hence Statement 3 is incorrect.

Donbas Region in Russian Map

Page 3: BYJU'S UPSC Monthly Magazine March 2022 Answer Key

Image source: CBC News

Q4. Which of the given statements with respect to SWIFT is/are correct?

1. It is a messaging network used by banks and financial institutions globally for quick and faultless

exchange of information pertaining to financial transactions.

2. It is a global member-owned cooperative that transfers funds between a domestic Indian bank and an overseas bank.

3. SWIFT does not hold any funds or securities.

4. SWIFT does not manage client accounts.

Options:

a. 2, 3 and 4 only b. 1 and 3 only

c. 1, 3 and 4 only

d. 1, 2, 3 and 4

Answer: c

Explanation:

SWIFT is a secure financial message carrier that can avoid fraudulent transactions. In other words, it

transports messages from one bank to another bank.

SWIFT is a global messaging software company headquartered in Brussels, Belgium. Hence Statement 1

is correct.

Page 4: BYJU'S UPSC Monthly Magazine March 2022 Answer Key

It is a global member-owned cooperative. SWIFT is the world’s international payment network used by

banks and other financial systems to quickly, accurately, and securely send and receive information. Hence Statement 2 is incorrect.

SWIFT is solely a carrier of messages. It does not hold funds nor does it manage accounts on behalf of

customers, nor does it store financial information on an ongoing basis. As a data carrier, SWIFT

transports messages between two financial institutions. Hence statements 3 and 4 are correct.

Q5. Which one of the following books of ancient India has the love story of the son of the founder of the Sunga

dynasty? [UPSC 2016]

a. Swapnavasavadatta

b. Malavikagnimitra

c. Meghadoota d. Ratnavali

Answer: b

Explanation:

Malavikagnimitra is a Sanskrit play by Kalidasa which tells the story of the love of Agnimitra.

Agnimitra was the Shunga emperor at Vidisha who falls in love with the picture of an exiled servant girl

named Malavika.

When the queen discovers her husband’s passion for this girl, she becomes infuriated and has Malavika imprisoned but as fate would have it, in the end, she is discovered to be of royal birth and is accepted as

one of his queens.

Hence option B is correct.

Q6. Consider the following statements with respect to Bardoli Satyagraha:

1. It is a movement in the independence struggle led by Vittalbhai J Patel for the farmers of Bardoli.

2. The movement was launched against the unjust raising of taxes by the British Government. 3. Mahatma Gandhi supported this movement through his writings in Young India.

Which of the given statements is/are correct?

a. 1 and 2 only

b. 1, 2 and 3 c. 2 and 3 only

d. 2 only

Answer: c

Explanation:

The Bardoli Satyagraha, 1928 was a movement in the independence struggle led by Sardar Vallabhai Patel for the farmers of Bardoli against the unjust raising of taxes. Hence statement 1 is not correct.

The Bardoli Taluk in modern-day Gujarat was hit by floods and famines in 1925, which adversely

affected crop yield. This affected the farmers financially.

Page 5: BYJU'S UPSC Monthly Magazine March 2022 Answer Key

Despite petitions and appeals from civic groups and farmers to review this unjust hike in tax rates in lieu

of the grave situation, the government decided to go ahead with tax collection. Hence statement 2 is correct.

In 1927, the local Congress Party published a report to show that the farmers could not carry the burden

of the enhanced assessment. But the authorities did not budge.

In January 1928, farmers in Bardoli invited Vallabhai Patel to launch the protest movement wherein all of

them resolved not to pay taxes.

Gandhiji also supported the movement through his writings in ‘Young India’ magazine. Hence statement 3 is correct.

Q7. Consider the following statements with respect to Bhakra Beas Management Board (BBMB):

1. It is a statutory body that is constituted under the Punjab Reorganisation Act, 1966.

2. It regulates the supply of water from Satluj, Ravi and Beas to the States of Punjab, Haryana and

Rajasthan. 3. It regulates the supply of power generated from Bhakra-Nangal and Beas Projects.

Which of the given statements is/are INCORRECT?

a. 1 only

b. 2 only

c. 1 and 3 only d. None of the above

Answer: d

Explanation:

The BBMB is a statutory body constituted under Section 79 and Section 80 of the Punjab Reorganisation

Act, 1966.Hence statement 1 is correct.

Functions of BBMB: o Regulating the supply of water from rivers Satluj, Ravi and Beas to the States of Punjab, Haryana

and Rajasthan. Hence statement 2 is correct.

o Regulating the supply of power generated from Bhakra-Nangal and Beas Projects. Hence statement 3 is correct.

Additional Functions:

o Construction of new Hydro Projects within and outside the BBMB System.

Q8. Which of the given statements with respect to the International Federation of Association Football (FIFA)

is/are correct?

1. It is the international governing body for futsal and beach soccer. 2. FIFA being the highest global governing body of football sets the rules of the game.

3. The FIFA Congress is the supreme legislative body of FIFA, in which each of the FIFA members has a

vote. 4. It is a member of the International Football Association Board.

Options:

a. 2 and 4 only

Page 6: BYJU'S UPSC Monthly Magazine March 2022 Answer Key

b. 2, 3 and 4 only c. 1, 3 and 4 only

d. 3 only

Answer: c

Explanation:

FIFA is also the international governing body for futsal (a kind of mini football played indoors on a hard court between two teams of five players each) and beach soccer (five-a-side, played on a beach). Hence

statement 1 is correct.

FIFA is the highest global governing body of football, the world’s most popular game. But it does not set

the rules of games. Hence statement 2 is not correct.

The FIFA Congress is the supreme legislative body of FIFA, in which each of the 211 members of the organization has a vote. Hence statement 3 is correct.

FIFA is a member of the International Football Association Board, which sets the rules of football, and it

applies and enforces those rules across all FIFA competitions. Hence Statement 4 is correct.

Q9. Which of the given statements is/are correct?

1. UNHCR is a UN Refugee Agency dedicated to saving lives, protecting the rights and building a better

future for refugees, forcibly displaced communities and stateless people. 2. UNHCR was created in 1950 to help millions of Europeans who had fled or lost their homes.

3. UNHCR is the only international organization to have won the Nobel Peace Prize more than once.

4. India is not a party to the UN Refugee Conventions.

Options:

a. 1, 2 and 4 only b. 3 and 4 only

c. 1 and 2 only

d. 1, 2, 3 and 4

Answer: a

Explanation:

UNHCR, the UN Refugee Agency, is an international organization working to save lives, safeguard the

rights and provide a better future for refugees. Hence statement 1 is correct.

The UNHCR was established in 1950 in the wake of the mass displacements caused due to the Second

World War in Europe. Hence statement 2 is correct.

UNHCR has received the Nobel Peace Prize twice, in 1954 and 1981, while the work of Comité international de la Croix Rouge (International Committee of the Red Cross) (ICRC) has been honoured

three times, in 1917, 1944 and 1963. Hence statement 3 is not correct.

The organisation works in 135 countries and in India, has offices in New Delhi and Chennai. It first

established its presence in India in 1981.

India is not a party to the UN Refugee Conventions. Hence statement 4 is correct.

Page 7: BYJU'S UPSC Monthly Magazine March 2022 Answer Key

Q10. What is ‘Greenhouse Gas Protocol’? [UPSC 2016]

a. It is an international accounting tool for government and business leaders to understand, quantify and

manage greenhouse gas emissions

b. It is an initiative of the United Nations to offer financial incentives to developing countries to reduce greenhouse gas emissions and to adopt eco-friendly technologies

c. It is an inter-governmental agreement ratified by all the member countries of the United Nations to reduce

greenhouse gas emissions to specified levels by the year 2022 d. It is one of the multilateral REDD+ initiatives hosted by the World Bank

Answer: a

Explanation:

GHG Protocol establishes comprehensive global standardized frameworks to measure and manage

greenhouse gas (GHG) emissions from private and public sector operations, value chains and mitigation actions.

GHG Protocol supplies the world’s most widely used greenhouse gas accounting standards.

Hence option ‘a’ is correct.

Q11. Consider the following statements:

1. World Wildlife Day is celebrated on the 3rd of March as on this day in 1973, the Convention on

International Trade in Endangered Species of Wild Fauna and Flora (CITES) was signed. 2. According to the WWF, the most trafficked mammal in the world is Pangolins.

3. Vaquita is the world’s rarest marine mammal and is on the edge of extinction.

Which of the statements given above is/are correct?

a. 1 only

b. 1 and 3 only c. 2 and 3 only

d. 1, 2 and 3

Answer: d

Explanation:

All the above statements are correct.

Hence option d is correct.

Q12. With respect to National Commission for Protection of Child Rights (NCPCR), which of the following

statements is/are correct?

1. It is an extra-constitutional body.

2. It functions under the Ministry of Home Affairs (MHA) of the central government.

3. A person who has held the office of the chairperson shall be eligible for re-nomination.

Options:

Page 8: BYJU'S UPSC Monthly Magazine March 2022 Answer Key

a. 1 and 2 only b. 2 and 3 only

c. 1 and 3 only

d. None

Answer: c

Explanation:

Statement 1 is correct. The National Commission for Protection of Child Rights (NCPCR) is a statutory

body established in 2007 under the Commissions for Protection of Child Rights (CPCR) Act, 2005.

Statement 2 is not correct. It functions under the Ministry of Women & Child Development of the central

government.

Statement 3 is correct. A person who has held the office of the chairperson shall be eligible for re-nomination.

Q13. Which of the following statements about Inter-governmental Panel on Climate Change (IPCC) is/are

correct?

1. The IPCC engages in scientific research.

2. The IPCC provides objective and comprehensive scientific information on anthropogenic climate change, including the natural, political, and economic impacts and risks, and possible response options.

Options:

a. 1 only

b. 2 only

c. Both d. None

Answer: b

Explanation:

Statement 1 is not correct. The IPCC does not conduct its own research but instead, it provides regular assessments of the scientific basis of climate change, its impacts and future risks, and options for

adaptation and mitigation.

Statement 2 is correct. The IPCC provides objective and comprehensive scientific information on

anthropogenic climate change, including the natural, political, and economic impacts and risks, and

possible response options.

Q14. Which of the following States are parties to Convention on Cluster Munitions?

1. France

2. New Zealand

3. United Kingdom

4. India 5. Ukraine

6. Russia

7. South Africa

Page 9: BYJU'S UPSC Monthly Magazine March 2022 Answer Key

Options:

a. 1, 2, 3 and 7 only

b. 1, 3, 4 and 7 only

c. 2, 3, 4, 5 and 6 only d. 2, 5 and 7 only

Answer: a

Explanation:

The Convention on Cluster Munitions is a United Nations-adopted legal instrument that prohibits all use, production, transfer and stockpiling of cluster munitions.

The convention was adopted in Dublin, Ireland in 2008, and was opened for signature in Oslo, Norway.

Currently, the convention has 110 State Parties and 13 Signatory States.

India has not signed the convention and is not a party to it. Other countries that are not parties are the US,

Russia, China, Pakistan and Israel.

Read more about Cluster Bombs.

Q15. Why is there a concern about copper smelting plants? (UPSC CSE 2021)

1. They may release lethal quantities of carbon monoxide into the environment 2. The copper slag can cause the leaching of some heavy metals into the environment

3. They may release sulphur dioxide as a pollutant

Select the correct answer using the code given below

a. 1 and 2 only

b. 2 and 3 only c. 1 and 3 only

d. 1, 2 and 3

Answer: b

Explanation:

Statement 1 is not correct. During the copper smelting process, initially, the Carbon combusts with

Oxygen in the air to produce carbon monoxide (CO). Later, the carbon monoxide reacts with ore and

releases carbon dioxide. Hence, it does not release a lethal quantity of CO into the atmosphere.

Statement 2 is correct. When copper slag remains on the soil, it may lead to leaching with the required

conditions present.

Statement 3 is correct. The copper smelting process may release sulphur dioxide.

Q16. Who among the following set up the ‘Balhatya Pratibandhak Griha’?

a. Raja Ram Mohan Roy

b. Ramabai Ranade

c. Tarabai Shinde

d. Savitribai Phule

Page 10: BYJU'S UPSC Monthly Magazine March 2022 Answer Key

Answer: d

Explanation:

Savitribai Phule together with her husband opened a care centre called Balhatya Pratibandhak Griha

(Child-killing Prohibition Home) for pregnant rape victims and helped deliver and save their children.

Hence option d is correct.

Q17. Consider the following Statements with respect to QUAD:

1. The idea of “QUAD” was first mooted by Japanese Prime Minister Shinzo Abe in 2007.

2. All the QUAD members are the members of Indian Ocean Rim Association (IORA).

3. All the QUAD members first participated in the Malabar Exercise in 2020.

Which of the given statements is/are INCORRECT?

a. 2 and 3 only b. 2 only

c. 3 only

d. 1 only

Answer: a

Explanation:

Statement 1 is correct. The idea of Quad was first mooted by Japanese Prime Minister Shinzo Abe in

2007.

Statement 2 is not correct. While India and Australia are members of Indian Ocean Rim Association

(IORA), the U.S.A. and Japan are dialogue partners.

Statement 3 is not correct. During the 2004 Tsunami, India conducted relief and rescue operations for itself and neighbouring countries and was joined by Japan, the US and Australia. The move was

paralleled by the joint military exercise ‘Malabar’ which was held at an unprecedented scale involving

India, Japan, Australia, Singapore and the US.

Q18. Consider the following statements with respect to the United Nations Environment Assembly (UNEA):

1. It is hosted by the UN Environment Programme (UNEP).

2. It is the world’s highest-level decision-making body on the environment.

3. It was created in 2012 during the Rio+20 negotiations.

Which of the given statements are INCORRECT?

a. 2 only b. 2 and 3 only

c. 1 only

d. None of the above

Answer: d

Explanation:

Page 11: BYJU'S UPSC Monthly Magazine March 2022 Answer Key

Statement 1 is correct. The UNEP’s governing body is called the United Nations Environment Assembly.

Statement 2 is correct. UNEA is said to be the world’s highest decision making body on the environment.

Statement 3 is correct. UNEA was formed in 2012 during the Rio+20 negotiations and is headed by a Bureau and its President.

Q19. Consider the following statements with respect to the SVAMITVA Scheme:

1. The scheme is piloted by the Ministry of Rural Development.

2. Under the scheme, residential land in villages will be measured using drones to create a non-disputable

record. 3. Under the scheme, property card for every property in the village will be prepared by states.

Which of the given statements is/are correct?

a. 1 and 3 only

b. 2 and 3 only

c. 1, 2 and 3 d. 2 only

Answer: b

Explanation:

Statement 1 is not correct. The Nodal Ministry for the SVAMITVA scheme is the Ministry of Panchayati Raj.

Statement 2 is correct. Drone Surveying technology and Continuously Operating Reference Station

(CORS) will be used for demarcation of rural abadi areas.

Statement 3 is correct. Under the scheme, SVAMITVA property cards for every property in the village

will be prepared by states.

Q20. With reference to the scholars/litterateurs of ancient India, consider the following statements:

1. Panini is associated with Pushyamitra Shunga 2. Amarasimha is associated with Harshavardhana

3. Kalidasa is associated with Chandra Gupta-II

Which of the statements given above is/are correct? [UPSC 2020]

a. 1 and 2 only

b. 2 and 3 only c. 3 only

d. 1, 2 and 3

Answer: c

Explanation:

Statement 1 is not correct. Patanajali, one of the greatest geniuses in Indian history, was a student of Panini.

Page 12: BYJU'S UPSC Monthly Magazine March 2022 Answer Key

o Patanjali is a contemporary of Pushyamitra Shunga.

Statement 2 is not correct. Amarasimha is associated with the Gupta age and Harshavardhana is a post-Gupta emperor.

Statement 3 is correct. Kalidasa belonged to the court of Chandra Gupta-II (Vikramaditya).

Q21. Which of the following is/are correctly matched?

Rescue Mission Name Country

1. Raahat Yemen 2. Maitri Nepal

3. Safe Homecoming Syria

4. Sukoon Lebanon

Options:

a. 1, 2 & 4 only b. 2, 3 & 4 only

c. 1 & 4 only

d. All of the above

Answer: a

Explanation:

Operation Raahat was an operation of the Indian Armed Forces to evacuate Indian citizens and foreign

nationals from Yemen during the 2015 military intervention by Saudi Arabia and its allies in that country

during the Yemeni Crisis. Hence pair 1 is correct

Operation Maitri (Operation Amity) was a rescue and relief operation in Nepal by the government of India and Indian armed forces in the aftermath of the April 2015 Nepal earthquake. Hence pair 2 is

correct

Operation Safe Homecoming was an operation launched by the Indian government in 2011, to evacuate

its citizens who were fleeing from the Libyan Civil War. The air-sea operation was conducted by the Indian Navy and Air India. Hence pair 3 is not correct

Operation Sukoon: It was an operation launched by the Indian Navy to evacuate Indian, Sri Lankan and

Nepalese nationals, as well as Lebanese nationals with Indian spouses, from the conflict zone during the

2006 Lebanon War. Hence pair 4 is correct

Q22. Consider the following statements with regards to IAEA (International Atomic Energy Agency):

1. In 2005, it was awarded the Nobel Peace Prize for their work for a safe and peaceful world. 2. The IAEA was created in 1957 in response to the deep fears and expectations generated by the

discoveries and diverse uses of nuclear technology.

3. It is one of the specialized agencies of the United Nations.

Choose the correct code:

a. 1 & 2 only b. 2 & 3 only

c. 1 & 3 only

Page 13: BYJU'S UPSC Monthly Magazine March 2022 Answer Key

d. All of the above

Answer: a

Explanation:

IAEA’s origins can be traced back to an address of the former US President Dwight Eisenhower to the

General Assembly of the United Nations in 1953.

The IAEA was awarded the Nobel Peace Prize in 2005. Hence Statement 1 is correct.

The address was known as ‘Atoms for Peace’ and this was the organization’s first name when it was

formally established in 1957. Hence Statement 2 is correct.

It is not a specialized agency of the United Nations. Hence Statement 3 is not correct.

Q23. Which of the following is/are a part of Literary work done by Sri Aurobindo:

1. An English newspaper called Bande Matram

2. Bases of Yoga

3. The Future Evolution of Man

4. The story of my experiments with Truth 5. Savitri: A Legend and a Symbol

6. Hour of God

Options:

a. All except 3 & 4 b. All except 1 & 5

c. All except 4

d. All except 2 & 6

Answer: c

Explanation:

Some of the literary works of Aurobindo are as follows: –

o Bases of Yoga

o Bhagavad Gita and Its Message o The Future Evolution of Mant

o Rebirth and Karma

o Savitri: A Legend and a Symbol

o Hour of God

The Story of My Experiments with Truth is the autobiography of Mahatma Gandhi, covering his life from early childhood through to 1921. Hence Statement 4 is not correct

Q24. _________________ is produced by electrolysis of water using renewable energy (like Solar, Wind) and has a lower carbon footprint.

a. Gray Hydrogen

b. Green Hydrogen c. Blue Hydrogen

d. Brown Hydrogen

Page 14: BYJU'S UPSC Monthly Magazine March 2022 Answer Key

Answer: b

Explanation:

Green hydrogen is produced by splitting water into hydrogen and oxygen using renewable electricity.

This is a unique process compared to the process of producing gray and blue hydrogen.

Hence option B is correct

Q25. With reference to the history of India, "Ulgulan" or the Great Tumult is the description of which of the

following events?

a. The Revolt of 1857

b. The Mappila Rebellion of 1921 c. The Indigo Revolt of 1859 – 60

d. Birsa Munda’s Revolt of 1899-1900

Answer: d

Explanation:

Munda Rebellion is one of the prominent 19th century tribal rebellions in the subcontinent.

Birsa Munda led this movement in the region south of Ranchi in 1899-1900.

The ulgulan, meaning ‘Great Tumult’, sought to establish Munda Raj and independence. Therefore, the correct answer is (d).

Q26. Which of the following statements with respect to Financial Action Task Force (FATF) is/are incorrect?

1. It was established in 1989 on the initiative of the Group of Twenty (G20) to examine and develop

measures to combat money laundering.

2. The FATF Secretariat is located at the OECD Headquarters in Paris. 3. The FATF currently comprises 37 member jurisdictions and 2 regional organizations, representing most

major financial centers in all parts of the globe.

Options:

a. 1 only b. 2 and 3 only

c. 3 only

d. None

Answer: a

Explanation:

The Financial Action Task Force (FATF) is an inter-governmental organization established to set

standards measures for combating money laundering, terrorist financing and other related threats to the

integrity of the international financial system.

It was formed in 1989 by the G7 Summit which was held in Paris. Hence Statement 1 is not correct

Page 15: BYJU'S UPSC Monthly Magazine March 2022 Answer Key

The FATF is really a policy-making body that works with governments to bring about national legislation

and regulatory reforms in these areas. Hence Statement 2 is correct

The FATF currently comprises 37 member jurisdictions and 2 regional organisations, representing most major financial centres in all parts of the globe. Hence Statement 3 is correct

Q27. With respect to Northern River Terrapin, which of the following statements is/are correct?

1. They are one of the largest sea turtle species and also one of the most migratory, crossing both the

Atlantic and Pacific Oceans

2. It is listed on the IUCN Red List as a Critically Endangered species

Options:

a. 1 only

b. 2 only

c. Both

d. None

Answer: b

Explanation:

Leatherback turtles are the largest sea turtle species and also one of the most migratory, crossing both the

Atlantic and Pacific Oceans. Hence Statement 1 is not correct

The northern river terrapin (Batagur baska) is a species of riverine turtle native to Southeast Asia. It is classified as Critically Endangered by the IUCN and considered extinct in much of its former range.

Hence Statement 2 is correct

Q28. Consider the following statements:

1. It is a carbohydrate-enriched coating that covers the outside of many eukaryotic cells and prokaryotic

cells, particularly bacteria. 2. It helps the pathogen to evade the immune system

The above statements best describe:

a. Pili

b. Flagella

c. Glycocalyx d. Endospore

Answer: c

Explanation:

Glycocalyx is a highly charged layer of membrane-bound biological macromolecules attached to a cell

membrane. This layer functions as a barrier between a cell and its surrounding.

The glycocalyx is a carbohydrate-enriched coating that covers the outside of many eukaryotic cells and prokaryotic cells, particularly bacteria .

Page 16: BYJU'S UPSC Monthly Magazine March 2022 Answer Key

The presence of a glycocalyx increases the effective diameter of a bacterium and also covers up

components of the bacterium that the immune system would detect and be stimulated by. It also helps the pathogen to evade the immune system.

Hence Option C is correct

Q29. Which of the following are correctly matched?

1. AH-64E Apache attack helicopter: USA

2. CH-47 Chinook Helicopters: France 3. Ml-35 helicopters: Russia

4. Pole Star: UK

Options:

a. 1, 2 and 3 only

b. 1 and 3 only c. 2 and 4 only

d. 1, 3 and 4 only

Answer: b

Explanation:

The Boeing AH-64 Apache is an American twin-turboshaft attack helicopter with a tailwheel-type

landing gear arrangement and a tandem cockpit for a crew of two. It features a nose-mounted sensor suite

for target acquisition and night vision systems. Hence Pair 1 is correct

The Boeing CH-47 Chinook is a tandem rotor helicopter developed by American rotorcraft company

Vertol and manufactured by Boeing Vertol. The Chinook is a heavy-lift helicopter that is among the heaviest lifting Western helicopters. Hence Pair 2 is not correct

The Mi-35M is a multi-role combat helicopter manufactured by Rostvertol, a subsidiary of Russian

Helicopters. It is an export variant of the Mi-24 Hind attack helicopter. Hence Pair 3 is correct

The ALH (Dhruv) (“Pole Star”) is a multi-role helicopter developed and manufactured by India’s

Hindustan Aeronautics Limited (HAL). Military versions inducted into the ARMY are for transport, utility, reconnaissance and Casualty -Evacuation roles. Hence Pair 4 is not correct

Q30. With reference to the Indian economy, demand-pull inflation can be caused/increased by which of the following?

1. Expansionary policies

2. Fiscal stimulus 3. Inflation-indexing wages

4. Higher purchasing power

5. Rising interest rates Select the correct answer using the code given below.

a. 1, 2 and 4 only

b. 3, 4 and 5 only c. 1, 2, 3 and 5 only

d. 1, 2, 3, 4 and 5

Page 17: BYJU'S UPSC Monthly Magazine March 2022 Answer Key

Answer: a

Explanation:

The rising pressure on prices that accompanies a supply shortage, which economists define as “too many dollars chasing too few things,” is known as demand-pull inflation.

Expansionary policies: Money in the market rises when the government spends more freely. Demand for

goods rises resulting in demand-pull inflation.

Fiscal Stimulus: Also raises the amount of money in circulation, which stimulates demand for products

and fuels demand-pull inflation.

Higher Purchasing Power: Consumers who make more money are more confident and spend more. As a result, there is an increase in demand, hence inflation is caused by consumer demand

Wages that are inflation-indexed and interest rates that are growing do not raise or produce demand-pull

inflation.

Hence Option A is correct

Q31. Which of the following National Waterways is/are correctly matched?

National Waterway River

1. NW-1 Ganga-Bhagirathi-Hooghly River System

2. NW-2 Brahmaputra River

3. NW-3 Krishna and Godavari 4. NW-10 Amba River

Options:

a. 1, 2 and 3 only

b. 3 and 4 only

c. 1, 2 and 4 only d. 1, 2, 3 and 4

Answer: c

Explanation:

The National Waterways Act came into effect in 2016.

It proposed 106 additional National Waterways and merged 5 existing Acts which were declared the 5 National Waterways.

As a result, 106 new waterways were identified by IWAI and intimated to MoS.

Sl. No. NW

Number

River System Route Length (in

km)

Locations Established

1 NW – 1 Ganga-

Bhagirathi-Hooghly

Prayagraj –

Haldia

1620 Uttar

Pradesh, Bihar,

Jharkhand,

1986

Page 18: BYJU'S UPSC Monthly Magazine March 2022 Answer Key

West Bengal

2 NW – 2 Brahmaputra Sadiya-

Dhubri

891 Assam 1988

3 NW – 3 West Coast Canal,

Champakara

Canal, and

Udyogamandal Canal

Kottapuram – Kollam

205 Kerala

10 NW – 111 Zuari–

Sanvordem Bridge to

Mormugao

Port

50 Goa 10

Hence Option C is correct.

Q32. The Central Industrial Security Force (CISF) provides security cover to

1. Nuclear Installations

2. Space Establishments

3. Private corporate establishments

4. VIP’s nominated by the Home Ministry

Options:

a. 1 and 2 only

b. 1, 2 and 3 only

c. 3 and 4 only

d. 1, 2, 3 and 4

Answer: d

Explanation:

Central Industrial Security Force (CISF):

The Central Industrial Security Force came into existence in 1969 to provide integrated security cover to

certain sensitive public sector undertakings with a strength of only three battalions.

The CISF security umbrella includes India’s most critical infrastructure facilities like nuclear

installations, space establishments, airports, seaports, Private corporate establishments, power plants etc.

In addition, the CISF also protects important government buildings, iconic heritage monuments and the

Delhi Metro.

Hence Option D is correct.

Q33. Consider the following statements with respect to HANSA- NG:

1. It is Unmanned Aerial Vehicle (UAV) developed by Aeronautical Development Establishment

2. It will replace/supplement the Heron UAVs in service with the Indian armed forces

Page 19: BYJU'S UPSC Monthly Magazine March 2022 Answer Key

3. It can be deployed at high altitude areas like Ladakh and in harsh environments for day-and-night surveillance by infantry soldiers and Special Forces

Which of the above statements is/are correct?

a. 1 only

b. 2 and 3 only

c. None d. 1, 2 and 3

Answer: c

Explanation:

HANSA-New Generation is designed and developed by the Council of Scientific and Industrial Research-

National Aerospace Laboratories (CSIR-NAL).

It is one of the most advanced flying trainers designed to meet the need of flying clubs in India for trainer aircraft.

It is an ideal aircraft for commercial pilot licensing due to its low cost and low fuel

Hence all the statements are incorrect.

Q34. Mariupol, a major port city recently seen in the news, is part of which country?

a. Romania

b. Poland c. Belarus

d. Ukraine

Answer: d

Explanation:

Mariupol is a city of regional significance in south eastern Ukraine, situated on the north coast of the Sea of Azov at the mouth of the Kalmius river, in the Pryazovia region.

It is the tenth-largest city in Ukraine, and the second largest in Donetsk Oblast.

Hence Option D is correct.

Q35. Consider the following statements in respect of the Laureus World Sports Award which was instituted in the year 2000: [PYQ 2021]

1. American golfer Tiger Woods was the first winner of this award.

2. The award has been received mostly by ‘Formula One’ Players so far. 3. Roger Federer received this award the maximum number of times compared to others.

Which of the above statements are correct?

a. 1 and 2 only

b. 2 and 3 only

c. 1 and 3 only

d. 1, 2 and 3

Page 20: BYJU'S UPSC Monthly Magazine March 2022 Answer Key

Answer: c

Explanation:

Since 2000, the Laureus World Sports Awards have included a number of accolades given by the

Academy at their discretion.

The American golfer was the first winner of this award. Hence, Statement 1 is correct.

The award was received mostly by ‘Tennis players’ (11 awards) where ‘Formula one’ and ‘athletes’ have won four. Hence statement 2 is not correct.

Roger Federer received this award the maximum number of times (5 times) compared to others. Hence,

Statement 3 is correct.

Q36. Consider the following statements with regards to the PM Shram Yogi Maan-Dhan Yojana:

1. The scheme covers unorganised workers whose monthly income is Rs 15,000/per month or less. 2. The Subscriber should belong to the entry age group of 18-40 years.

3. They should not be covered under New Pension Scheme (NPS), Employees’ State Insurance Corporation

(ESIC) scheme or Employees’ Provident Fund Organisation (EPFO).

4. He/She should be an income taxpayer.

Options:

a. 1, 2 & 3 only

b. 2, 3 & 4 only

c. 1 & 4 only d. All of the above

Answer: a

Explanation:

Pradhan Mantri Shram Yogi Maan-Dhan (PM-SYM) was launched by Prime Minister Narendra Modi

under the Ministry of Labour and Employment in February 2019.

To be eligible under the Pradhan Mantri Shram Yogi Maan Dhan, the candidate should fulfil the below-mentioned criteria:

o He/she should have a monthly income of Rs. 15,000 or below. Hence Statement 1 is correct.

o He/she should be an unorganized worker (UW) aged between 18 years to 40 years. Hence Statement 2 is correct.

o He/she should possess an Aadhar card along with a Savings Bank Account/Jan Dhan account

number with IFSC.

Any individual who is engaged in the Organised Sector (membership of EPF/NPS/ESIC) will not be

eligible to apply for the PM-SYM Scheme. Hence Statement 3 is correct.

Any individual who is an income taxpayer will not be eligible to apply for the PM-SYM Scheme. Hence Statement 4 is not correct.

Q37. Consider the following statements with regards to ‘Equalisation Levy’:

1. It is aimed at taxing foreign companies which have a significant local client base in India but are billing

them through their offshore units, effectively escaping the country’s tax system.

Page 21: BYJU'S UPSC Monthly Magazine March 2022 Answer Key

2. For EL to be imposed, the annual payment which is made to a service provider should be more than 10 Lakh rupees in one financial year.

3. The amendments to the Finance Act, 2020 had expanded the ambit of the equalisation levy for non-

resident e-commerce operators involved in the supply of services, including the online sale of goods and

provision of services, with the levy at the rate of 2% effective April 1, 2020.

Choose the correct code:

a. 1 & 2 only

b. 2 & 3 only

c. 1 & 3 only d. All of the above

Answer: c

Explanation:

Equalization levy is an extremely vital tax system that enables enterprises to regulate their business

models following the existing conventions.

Equalization levy is aimed at taxing foreign companies which have a significant local client base in India but are billing them through their offshore units, effectively escaping the country’s tax system. Hence

Statement 1 is correct.

According to this upgraded form of EL, any non-resident digital operator who is providing e-commerce

services is liable to be taxed at the rate of 2 per cent.

The payment should be made to a non-resident service provider; the annual payment which is made to a service provider should be more than 1 Lakh (Not 10 Lakh) in one financial year. Hence Statement 2 is

not correct.

India’s Finance Act 2020 (FA 2020) expanded the scope of the digital tax levy known as the Equalization

Levy (EL) to cover “e-commerce supplies or services.” With effect from 1 April 2020, EL is chargeable

at the rate of 2% on consideration received or receivable by nonresidents who operate digital businesses targeting, among others, the Indian market. Hence Statement 3 is correct.

Q38. Consider the following statements with regards to the Rajya Sabha elections:

1. The elections take place through Proportional Representation with the Single Transferable Vote

methodology. 2. Out of the 8 union territories, only Delhi, Puducherry and Jammu and Kashmir have representation in

Rajya Sabha.

3. All states have at least some representation in the Rajya Sabha.

Choose the correct code:

a. 1 & 2 only b. 2 & 3 only

c. 1 & 3 only

d. All of the above

Answer: d

Explanation:

Page 22: BYJU'S UPSC Monthly Magazine March 2022 Answer Key

Voting in Rajya Sabha is done by a single transferable vote, as the election is held on the principle of proportional representation. In other words, a bloc of MPs belonging to one or more parties can elect a

member of their choice if they have the requisite numbers. Hence Statement 1 is correct.

In total eight members are elected to Rajya Sabha from the Union territories (3 from Delhi, 1 from

Puducherry and 4 from Jammu & Kashmir). Other Union territories are not represented in the Rajya

Sabha. Hence Statement 2 is correct.

The Fourth Schedule to the Constitution provides for the allocation of seats to the States and Union Territories in Rajya Sabha. The allocation of seats is made on the basis of the population of each State.

All states have at least some representation in the Rajya Sabha. Hence Statement 3 is correct.

Q39. A situation of slow economic growth and relatively high unemployment is best defined as

a. Stagflation b. Inflation

c. Deflation

d. Galloping inflation

Answer: a

Explanation:

Stagflation refers to an economy that is experiencing a simultaneous increase in inflation and stagnation of

economic output. Stagflation was first recognized during the 1970s when many developed economies experienced rapid inflation and high unemployment as a result of an oil shock. The prevailing economic theory at the time

could not easily explain how stagflation could occur. Hence Option ‘a’ is correct.

Q40. With reference to the book "Desher Katha” written by Sakharam Ganesh Deuskar during the freedom struggle, consider the following statements:

1. It warned against the Colonial State’s hypnotic conquest of the mind. 2. It inspired the performance of swadeshi street plays and folk songs.

3. The use of ‘desh’ by Deuskar was in the specific context of the region of Bengal.

Which of the statements given above are correct? [UPSC 2020]

a. 1 and 2 only

b. 2 and 3 only c. 1 and 3 only

d. 1, 2 and 3

Answer: a

Explanation:

Only statements 1 and 2 are correct. Hence Option ‘a’ is correct.

Q41. Consider the following statements with regards to Rat Hole Mining:

Page 23: BYJU'S UPSC Monthly Magazine March 2022 Answer Key

1. Rat hole mining involves digging of very small tunnels, usually only 3-4 feet high, which workers (often children) enter and extract coal.

2. The National Green Tribunal (NGT) banned it in 2014, on grounds of it being unscientific and unsafe for

workers.

3. Since the coal seam is extremely thin in Meghalaya, no other method would be economically viable.

Choose the correct code:

a. 1 & 3 only

b. 2 & 3 only

c. 1 & 2 only d. All of the above

Answer: d

Explanation:

Commercial open mining is not practised in the North-Eastern regions of Assam and Meghalaya because

of the terrain’s unsuitability as well as the nature of coal deposits. The coal found in North-East contains lots of sulphur and this type of coal is categorized as bad quality of coal. Here, rat hole mining is

prevalent. A rat-hole mine involves digging of very small tunnels, usually only 3-4 feet deep, in which

workers, more often children, enter and extract coal.

Since the coal seam is extremely thin in Meghalaya, no other method would be economically viable in such areas.

The National Green Tribunal (NGT) banned it in 2014, and retained the ban in 2015, on grounds of it

being unscientific and unsafe for workers.

Q42. Consider the following statements with regards to BBIN Motor Vehicle Agreement:

1. The Bangladesh, Bhutan, India, Nepal (BBIN) Initiative is a sub-regional architecture of countries in

Eastern South Asia. 2. The BBIN Motor Vehicles Agreement was signed in 2015 at the BBIN transport ministers meeting in

Thimpu, Bhutan.

3. Apart from Nepal, all other nations have ratified the agreement already.

Choose the correct code:

a. 1 & 2 only b. 2 & 3 only

c. 1 & 3 only

d. All of the above

Answer: a

Explanation:

The BBIN Motor Vehicles Agreement (MVA) was signed in 2015 at the BBIN transport ministers

meeting in Thimpu, Bhutan to ensure the free flow of goods and people between them.

Page 24: BYJU'S UPSC Monthly Magazine March 2022 Answer Key

The agreement would permit the member states to ply their vehicles in each other’s territory for

transportation of cargo and passengers. Cargo vehicles will be able to enter any of the four nations without the need for trans-shipment of goods from one country’s truck to another’s at the border.

Notably, each vehicle would require an electronic permit to enter another country’s territory, and border

security arrangements between nations’ borders would also remain.

Though the original BBIN MVA was signed by all four countries, after objections in Bhutan over

sustainability and environmental concerns, the Bhutanese Parliament decided not to endorse the plan.

Q43. Consider the following statements:

1. Under the present rules for getting a PAN card, an applicant can furnish only their mother’s name, without giving their father’s name, if she is a single parent.

2. Under the Hindu Minority and Guardianship Act, (HMGA) 1956, the natural guardian of a Hindu minor

in respect of the minor’s person or property “is the father, and after him, the mother”. 3. The Muslim Personal Law (Shariat) Application Act, 1937 says that the Shariat or the religious law will

apply in case of guardianship according to which the father is the natural guardian.

Choose the correct code:

a. 1 & 2 only

b. 2 & 3 only c. 1 & 3 only

d. All of the above

Answer: d

Explanation:

Under the new Income Tax Rules, giving the father’s name is optional but only for those with single mothers.

Indian laws accord superiority to the father in case of guardianship of a minor. This is observed in the

Hindu Minority and Guardianship Act, (HMGA) 1956, as well as the Muslim Personal Law (Shariat)

Application Act, 1937. Both these acts recognize the father as the natural guardian of the child even as they provide the custody of a young child to a mother.

Q44. Which of the following statements about NATO (North Atlantic Treaty Organization) is not correct?

a. NATO constitutes a system of collective security, whereby its independent member states agree to mutual

defense in response to an attack by any external party.

b. The most recent member state to be added to NATO was Moldova on 27 March 2020. c. The September 11 attacks in the United States caused NATO to invoke Article 5 of the NATO Charter for

the first time in the organization’s history.

d. Article 5 of the treaty commits each member state to consider an armed attack against one member state, in Europe or North America, to be an armed attack against them all.

Answer: b

Explanation:

Page 25: BYJU'S UPSC Monthly Magazine March 2022 Answer Key

Bulgaria, Estonia, Latvia, Lithuania, Romania, Slovakia, and Slovenia joined in 2004, Albania and

Croatia in 2009, Montenegro in 2017, and North Macedonia in 2020, taking the membership of the NATO alliance to 30.

Q45. In which one of the following groups are all the four countries members of G20? [UPSC 2020]

a. Argentina, Mexico, South Africa and Turkey

b. Australia, Canada, Malaysia and New Zealand

c. Brazil, Iran, Saudi Arabia and Vietnam d. Indonesia, Japan, Singapore and South Korea

Answer: a

Explanation:

G20 is an international forum for global economic cooperation.

It has 20 members, and these countries are as follows:

o Argentina, Australia, Brazil, Canada, China, France, Germany, India, Indonesia, Italy, Japan, Republic of Korea, Mexico, Russia, Saudi Arabia, South Africa, Turkey, United Kingdom,

United States and the European Union.

Q46. Sarlamkai, Chheih Lam and Cheraw are the popular dance forms of which of these states?

a. Assam b. Mizoram

c. Sikkim

d. Nagaland

Answer: b

Explanation:

Sarlamkai is a dance form originating from the Pawi and Mara communities in the southern part of

Mizoram. In older days when the different tribes were constantly at war with each other, a ceremony to

deride the vanquished beheaded skull of the enemy was usually held by the victor.

Chheihlam is a dance that embodies the spirit of joy and exhilaration. It is performed to the accompaniment of a song called ‘Chheih hla’. People squat around in a circle on the floor, sing to the beat

of a drum or bamboo tube while a pair of dancers stand in the middle, recite the song and dance along

with the music.

Cheraw is a very old traditional dance of the Mizos. It is believed that the dance had already existed way

back in the 1st Century A.D., while the Mizos were still somewhere in the Yunan Province of China, before their migration into the Chin Hills in the 13th Century A.D., and eventually to the present

Mizoram. In this dance form, men sitting face to face on the ground tap long pairs of horizontal and cross

bamboo staves open and close in rhythmic beats. Girls in colourful Mizo costumes of ‘Puanchei’, ‘Kawrchei’, Vakiria’ and ‘Thihna’ dance in and out between the beats of bamboo. This dance is now

performed on almost all festive occasions.

Context:

Page 26: BYJU'S UPSC Monthly Magazine March 2022 Answer Key

The Vice President witnessed traditional dances of Mizoram Sarlamkai, Chheih Lam, and Cheraw, among

other cultural performances during his visit to the state.

Q47. Which of the given statements with respect to National Investment and Infrastructure Fund (NIIF) is/are

correct?

1. The National Investment and Infrastructure Fund (NIIF) is India’s first Sovereign Wealth Fund (SWF).

2. It is a collaborative investment platform for international and Indian investors anchored by the

Government of India. 3. NIIF Master Funds primarily invest in operating assets in core infrastructure sectors such as transportation

and energy.

Options:

a. 2 and 3 only

b. 2 only c. 1, 2 and 3

d. None of the above

Answer: c

Explanation:

The National Infrastructure and Investment Fund (NIIF) is India’s first-ever sovereign wealth fund

(SWF). The state-owned fund was set up by the Indian Government in the year 2015.

It is meant to be a collaborative investment platform for international and Indian investors.

The primary goal of setting up NIIF was to optimise the economic impact largely through investing in

infrastructure-related projects.

NIIF Limited manages over USD 4.5 billion of equity capital commitments across its three funds –

Master Fund, Fund of Funds and Strategic Opportunities Fund, each with its distinct investment strategy. NIIF Master Fund primarily invests in operating assets in core infrastructure sectors such as transportation

and energy. NIIF Fund of Funds invests in funds managed by best-in-class fund managers focused on

some of the most dynamic sectors in India such as climate infrastructure, middle-income & affordable housing, digital consumer platforms and other allied sectors. NIIF Strategic Opportunities Fund is a

Private Equity fund that aims to build scalable businesses across a range of opportunity long but capital

short sectors.

Q48. Merapi volcano, recently seen in News, is located in

a. Philippines b. Japan

c. Indonesia

d. Turkey

Answer: c

Explanation:

Mount Merapi is an active stratovolcano located in Indonesia

Page 27: BYJU'S UPSC Monthly Magazine March 2022 Answer Key

Q49. Arrange the following from North to South:

1. Pangong Tso Lake

2. Galwan Valley

3. Gogra Hot Springs

Options:

a. 2, 3, 1

b. 2, 1, 3

c. 3, 1, 2 d. 1, 3, 2

Answer: a

Explanation:

Source: The Quint

Q50. If a particular plant species is placed under Schedule VI of The Wildlife Protection Act, 1972, what is the

implication? [UPSC 2020]

a. A license is required to cultivate that plant.

b. Such a plant cannot be cultivated under any circumstances. c. It is a Genetically Modified crop plant.

d. Such a plant is invasive and harmful to the ecosystem

Answer: a

Explanation:

If a particular plant species is placed under Schedule VI of the Wildlife Protection Act, 1972, it means that a license is required to cultivate that plant.

Page 28: BYJU'S UPSC Monthly Magazine March 2022 Answer Key

Q51.With respect to India's military exercises, which of the following is/are matched correctly?

Military Exercise Country

1. Ajeya Warrior USA

2. Dharma Guardian Japan

3. Indra Indonesia 4. Yudh Abhyas UK

Options:

a. 1, 2 and 3 only

b. 2 only

c. 3 and 4 only d. 1, 2, 3 and 4

Answer: b

Explanation:

Ajeya Warrior: UK

Indra: Russia

Yudh Abhyas: US

Q52. Consider the following statements with respect to National Youth Parliament Festival:

1. It is organized by the Ministry of Parliamentary Affairs. 2. The objective of the National Youth Parliament Festival is to hear the voice of youth between 18 and less

than 25 years of age.

Which of the statements given above is/are correct?

a. 1 only

b. 2 only c. Both

d. None

Answer: b

Explanation:

The National Youth Parliament Festival (NYPF) has been organised jointly by the Lok Sabha Secretariat and the Ministry of Youth Affairs and Sports.

Q53. Which of the following statements is/are correct with respect to Kudankulam Nuclear Power Project

(KKNPP)?

1. It is a Civilian Nuclear Power Plant, built by India in joint collaboration with Russia.

2. It is the largest nuclear power station in India. 3. India is the world’s third-largest producer of nuclear power.

Page 29: BYJU'S UPSC Monthly Magazine March 2022 Answer Key

Options:

a. 1 and 2 only

b. 2 and 3 only c. 1 and 3 only

d. 1, 2 and 3

Answer: a

Explanation:

United States, China, France and Russia rank among the largest producers of nuclear energy in the world.

As per available data, India does not come even in the top ten nuclear energy producers currently.

Q54. Consider the following statements with respect to Cabinet committees:

1. Cabinet Committees find a mention in the Constitution.

2. All committees except Cabinet Committee on Accommodation and Cabinet Committee on Parliamentary Affairs are headed by Prime Minister.

Which of the statements given above is/are correct?

a. 1 only

b. 2 only

c. Both d. None

Answer: b

Explanation:

The cabinet committees are an extra-constitutional body, which means they are not mentioned in the

Constitution.

The Prime Minister sets up different cabinet committees with selected members of the Cabinet and assigns specific functions to these committees.

All committees except Cabinet Committee on Accommodation and Cabinet Committee on Parliamentary

Affairs are headed by Prime Minister.

Q55. At the national level, which ministry is the nodal agency to ensure effective implementation of the

Scheduled Tribes and Other Traditional Forest Dwellers (Recognition of Forest Rights) Act, 2006? (UPSC-2021)

a. Ministry of Environment, Forest and Climate Change

b. Ministry of Panchayati Raj

c. Ministry of Rural Development

d. Ministry of Tribal Affairs

Answer: d

Explanation:

Page 30: BYJU'S UPSC Monthly Magazine March 2022 Answer Key

At the national level, the Ministry of Tribal Affairs is the nodal agency to ensure implementation of the Scheduled Tribes and Other Traditional Forest Dwellers (Recognition of Forest Rights) Act, 2006.

Q56. Rashtriya Garima Abhiyan is a national campaign to

a. Rehabilitate children in conflict with the law

b. Abolish human trafficking and slavery

c. Eliminate and prevent child labour d. Eradicate manual scavenging

Answer: d

Explanation:

Rashtriya Garima Abhiyan is a National Campaign for Dignity and Eradication of Manual Scavenging.

It aims to eradicate the inhuman practice of manual scavenging and comprehensive rehabilitation of

manual scavengers in India.

Hence Option D is correct.

Q57. Which of the following subjects are listed under the Concurrent List?

1. Education

2. Forest

3. Public Health and Sanitation 4. Relief of the disabled and unemployable

5. Taxes on luxuries, including taxes on entertainments

Options:

a. 1 and 2 only

b. 3, 4 and 5 only c. 1, 2, 4 and 5 only

d. 1, 2 and 4 only

Answer: a

Explanation:

Concurrent List has 52 subjects enumerated under it.

42nd amendment Act 1976 shifted below mentioned five subjects from State list to Concurrent List:

Education

Forests

o Protection of wild animals and birds

o Weights and measures and o Administration of justice, constitution and organization of all courts except the Supreme Court

and the High Courts

State List (List II) contains 66 items: state court fees, prisons, local government, public order, police,

public health and sanitation, hospitals and dispensaries, pilgrimages within India, intoxicating liquors, relief of disabled and unemployable, libraries, communications, agriculture, animal husbandry, water

supply, irrigation and canals, fisheries, road passenger tax and goods tax, taxes on luxuries, including

taxes on entertainments, capitation tax and others.

Page 31: BYJU'S UPSC Monthly Magazine March 2022 Answer Key

Hence Option A is correct.

Q58. Consider the following statements:

1. Nomination of members to Rajya Sabha was taken from the Irish Constitution.

2. President can nominate two members to the Lok Sabha from the Anglo-Indian Community.

Which of the statements given above is/are correct?

a. 1 only

b. 2 only

c. Both d. None

Answer: a

Explanation:

Nomination of members to Rajya Sabha, DPSP and method of election of President are the features

borrowed from the Irish Constitution. Hence statement 1 is correct.

The 104th amendment of the Indian Constitution removed the reserved seats for the Anglo-Indian community in the Lok Sabha and state assemblies. Hence statement 2 is not correct.

Q59. The inverse relationship between unemployment rate and inflation is depicted by

a. Kuznets Curve

b. Phillips Curve

c. Lorenz Curve d. Laffer Curve

Answer: b

Explanation:

The inverse relationship between the unemployment rate and inflation when graphically charted is called the Phillips curve. It is a single-equation economic model named after William Phillips.

The Phillips curve remains the primary framework for understanding and forecasting inflation used in

central banks.

Hence Option B is correct.

Q60. Which one of the following best defines the term ‘State’?

a. A community of persons permanently occupying a definite territory independent of external control and

possessing an organized government b. A politically organized people of a definite territory and possessing an authority to govern them, maintain

law and order, protect their natural rights and safeguard their means of sustenance

c. A number of persons who have been living in a definite territory for a very long time with their own

culture, tradition and government d. A society permanently living in a definite territory with a central authority, an executive responsible to

the central authority and an independent judiciary

Page 32: BYJU'S UPSC Monthly Magazine March 2022 Answer Key

Answer: a

Explanation:

The State is the politically organized people of a definite territory.

State is “a community of persons, more or less numerous, permanently occupying a definite portion of territory, independent, or nearly so, of external control, and possessing an organized government to which

the great body of inhabitants render habitual obedience.”

Hence Option A is correct.

Q61. Consider the following statements with respect to SARAS-3:

1. It is a radio telescope that is mounted on India’s first multi-wavelength astronomical observatory AstroSat.

2. It was built to detect extremely faint radio wave signals from the “Cosmic Dawn”.

3. SARAS-3 can be deployed on water bodies.

Which of the given statements is/are correct?

a. 1 and 2 only b. 2 and 3 only

c. 3 only

d. 1, 2 and 3

Answer: b

Explanation:

SARAS-3 is an indigenously built telescope that was developed by the researchers of Raman Research

Institute, an autonomous institute of the Department of Science and Technology. It is not mounted on

India’s first multi-wavelength astronomical observatory AstroSat. Hence Statement 1 is not correct.

SARAS-3 is a niche high-risk high-gain experimental effort to detect extremely faint radio wave signals from the depths of time, from our ‘Cosmic Dawn’. Hence Statement 2 is correct.

Its chief distinguishing characteristic is that it can be deployed on water bodies. The many layers of soil

were themselves a source of radio wave contamination for ground based telescopes. Given that the

purpose is to detect a highly elusive signal, water would be an ideal medium, the group reckoned, to make such a sensitive measurement .Hence Statement 3 is correct.

Q62. Consider the following statements with respect to NASA’s Perseverance rover:

1. It was designed as part of NASA’s Mars 2020 mission to explore the Jezero crater on Mars.

2. It uses the heat of plutonium’s radioactive decay for electric power.

3. The rover is carrying Ingenuity – the first helicopter to fly on Mars.

Which of the given statements is/are INCORRECT?

a. 1 and 3 only b. 3 only

c. 2 only

Page 33: BYJU'S UPSC Monthly Magazine March 2022 Answer Key

d. None of the above

Answer: d

Explanation:

The NASA Perseverance Rover is a part of the National Aeronautics and Space Administration’s (NASA)

Mars 2020 Mission. It is nicknamed ‘Percy’.Hence Statement 1 is correct.

Perseverance carries a radioisotope power system. This power system produces a dependable flow of

electricity using the heat of plutonium’s radioactive decay as its “fuel.”Hence Statement 2 is correct.

Perseverance also carried the Ingenuity helicopter (nicknamed Ginny). Ingenuity is a technology demonstration to test the first powered flight on Mars. Hence Statement 3 is correct.

Q63. Consider the following statements with respect to Pandavleni Caves:

1. These caves are early examples of Indian rock-cut architecture and represent Mahayana Buddhist

tradition.

2. The caves have the magnificent idols of Buddha and the popular Jain Tirthankaras such as Vrishabhdeo. 3. These caves house the paintings of the Gupta Period.

Which of the given statement/s is/are INCORRECT? a. 1 and 3 only

b. 1 only

c. 2 and 3 only d. 3 only

Answer: a

Explanation:

Nasik caves are ancient rock-cut caves built on the Trirashmi hill about 3004 feet above the sea, hence are

called Trirashmi Caves. They are also known as Pandavleni Caves.

The Pandavleni caves are a group of 24 caves which are Hinayana Faith Buddhist Caves.Hence Statement 1 is not correct.

The caves have the magnificent idols of Buddha and the popular Jain Tirthankaras such as Vrishabhdeo,

the icons of Bodhisattva, Veer Manibhadra Ji and Ambikadevi are also there. Hence Statement 2 is

correct.

Trirashmi caves are great examples of intricate carving and craftsmanship with outstanding sculptures.

The Inscriptions in the Nasik Caves belong to the Kshatrapa dynasty, Satavahana dynasty. Hence Statement 3 is not correct.

Q64. River Umiam flows through which of these states?

1. Assam 2. Meghalaya

3. Mizoram

4. Arunachal Pradesh

Options:

a. 1 and 2 only

b. 3 only

Page 34: BYJU'S UPSC Monthly Magazine March 2022 Answer Key

c. 4 only d. 2 only

Answer: a

Explanation:

The River Umiam takes its origin from the Shillong Peak, the highest point in Meghalaya .

The Kiling/Umiam flows through the states of Assam and Meghalaya. It is a north flowing river

originating from the northern slopes of the Khasi hill range.

The river passes through Mawphu, Thieddieng, Nongsteng areas and enters the River Umiam Shella Basin, crossing the international border into Bangladesh.

Hence Option A is correct.

Q65. In India, separation of judiciary from the executive is enjoined by

a. The Preamble of the Constitution

b. A Directive Principle of state policy

c. The Seventh schedule d. The conventional practice

Answer: b

Explanation:

Article 50 of Constitution of India is a directive principle of state policy. It gives a direction to the State to

keep Judiciary independent of the Executive, particularly in judicial appointments.

Hence option B is correct

Q66. Consider the following statements with respect to Lake Chilika:

1. Mangalajodi Bird Sanctuary is the core area of Chilika Lake. 2. It is Asia’s largest coastal lagoon.

3. Chilika Lake boasts the highest single lagoon population of Irrawaddy dolphins in the world.

Which of the given statement/s is/are correct?

a. 2 and 3 only b. 1 and 2 only

c. 3 only

d. 1, 2 and 3

Answer: a

Explanation:

Mangalajodi bird sanctuary in Odisha located on the northern edge of Chilka. Hence Statement 1 is not

correct

Lake Chilika is the largest brackish water lagoon in Asia and the second largest coastal lagoon in the

world. Hence Statement 2 is correct

Page 35: BYJU'S UPSC Monthly Magazine March 2022 Answer Key

This salt lake has the highest single lagoon population of Irrawaddy dolphins in the world. Tourists gather

around the year to enjoy sighting Irrawaddy dolphins near Satpada Island. Hence Statement 3 is correct

Q67. Which of the given statements is/are correct?

1. Palladium is used to coat electrodes that help control flow of electricity. 2. Palladium is used in making multilayer ceramic capacitors which are important to make smartphone

screens and power circuit breakers.

3. Ukraine is the world’s largest producer of palladium.

Options:

a. 3 only

b. 1 and 2 only

c. 1 and 3 only

d. 2 only

Answer: b

Explanation:

Most palladium is used in catalytic converters for cars. It is also used in jewellery and some dental fillings

and crowns. It is used in the electronics industry in ceramic capacitors, found in laptop computers and mobile phones. Hence Statement 1 is correct

In the electronics sector, most palladium is used for multi-layer ceramic (chip) capacitors (MLCC).

MLCCs store energy in electronic devices such as broadcasting equipment, mobile telephones,

computers, electronic lighting and high voltage circuits. Hence Statement 2 is correct

Russia is the leading global producer of palladium in recent years. Hence Statement 3 is not correct

Q68. Which of the given statements with respect to GenoMICC research project is/are INCORRECT?

1. It is a research study initiated by India that is exclusively aimed at mapping and identifying the genetic variation linked to severity of COVID-19.

2. It aims to undertake whole genome sequencing of thousands of individuals representing diverse ethnic

groups from India. 3. It is an initiative of the Council of Scientific and Industrial Research (CSIR).

Options:

a. 1 and 3 only

b. 1 only

c. 2 only d. 1, 2 and 3

Answer: d

Explanation:

GenOMICC (Genetics of Mortality in Critical Care)

Page 36: BYJU'S UPSC Monthly Magazine March 2022 Answer Key

It is a research study done by Scientists in the United Kingdom that brings together clinicians and

scientists from around the world to find the genetic factors that lead to critical illnesses.Hence Statement 1 is not correct

Researchers from the GenOMICC consortium, led by the University of Edinburgh in partnership with

Genomics England, sequenced the genomes of 7,491 patients from 224 intensive care units in the United

Kingdom. Hence Statement 2 and 3 are not correct

GenOMICC has been studying infections such as SARS (severe acute respiratory syndrome), MERS

(Middle East respiratory syndrome), flu and other forms of critical illness.

The overarching aim of genome association studies is to not only correlate genes but also design

treatments.

Q69. Which of the given statements is/are correct with respect to the National Pharmaceutical Pricing Authority? 1. In India, the prices of all drugs including branded and generic are regulated by the National

Pharmaceutical Pricing Authority.

2. It fixes the ceiling price of scheduled medicines in the first schedule of the Drugs (Prices Control) Order, 2013.

3. It prepares the National List of Essential Medicines (NLEM) annually.

Options: a. 1 and 3 only

b. 2 and 3 only

c. 1, 2 and 3 d. 1 and 2 only

Answer: d

Explanation:

The National Pharmaceutical Pricing Authority (NPPA) was set up as an attached office of the

Department of Chemicals and Petrochemicals on August 29, 1997.

It is responsible for regulating the prices of drugs in the country and ensuring their availability,

accessibility and affordability to every citizen of the country. Hence Statement 1 is correct

NPPA aims to implement and enforce the provisions of the Drugs Price Control Order (DPCO), 1995/2013 in accordance with the powers delegated to it. Hence Statement 2 is correct

The Ministry of Health and Family Welfare, Government of India prepares the first National List of

Essential Medicines of India. Hence Statement 3 is not correct

Q70. Which of the following phrases defines the nature of the ‘Hundi’ generally referred to in the sources of the

post-Harsha period?

a. An advisory issued by the king to his subordinates

b. A diary to be maintained for daily accounts

c. A bill of exchange

d. An order from the feudal lord to his subordinates

Answer: c

Explanation:

Harshavardhana was born in 590 AD to King Prabhakaravardhana of Sthaneshvara (Thanesar, Haryana).

He belonged to the Pushyabhuti also called the Vardhana dynasty.

Hiuen Tsang has given a very favorable account of king Harsha and his empire.

Page 37: BYJU'S UPSC Monthly Magazine March 2022 Answer Key

Hundis refer to financial instruments evolved on the Indian subcontinent used in trade and credit

transactions. They were used o as remittance instruments (to transfer funds from one place to another),

o as credit instruments (to borrow money [IOUs]),

o for trade transactions (as bills of exchange).

o Hence Option C is correct

Q71. Which of the following fundamental rights are available to both citizens and foreigners, except enemy

aliens? 1. Article 14

2. Article 19

3. Article 21 4. Article 25

5. Article 29

Options: a. 1, 3 and 4 only

b. 1, 2, and 5 only

c. 2, 3, 4 and 5 only d. 1, 2, 3, 4 and 5

Answer: a

Explanation:

Fundamental Rights available only to citizens and not to foreigners are:

Prohibition of discrimination on grounds of race, religion, caste, gender or place of birth (Article 15).

Equality of opportunity in matters of public employment (Article 16).

Protection of freedom of speech & expression (Article 19).

Protection of the culture, language and script of minorities (Article 29).

Right of minorities to establish and administer educational institutions (Article 30).

Hence Fundamental Rights available to both citizens and foreigners are Article 14, Article 21 and Article 25. Hence option A is correct.

Q72. Which of the following statements is/are correct?

1. Border infrastructure creation is largely the responsibility of the Ministry of Home Affairs.

2. Department of Border Management under the MHA deals with the management of borders, including coastal borders, strengthening of border guarding mechanisms and creation of related infrastructure,

border areas development, etc.

Options: a. 1 only

b. 2 only

c. Both 1 and 2 d. Neither 1 nor 2

Answer: c

Explanation:

Ministry of Home Affairs

Page 38: BYJU'S UPSC Monthly Magazine March 2022 Answer Key

The Ministry of Home Affairs looks after the creation of border infrastructure. Hence Statement 1 is

correct.

Department of Border Management under the MHA deals with the management of borders, including coastal borders, strengthening of border guarding mechanisms and creation of related infrastructure,

border areas development, etc. Hence Statement 2 is correct.

The Department is further organized into two divisions –

o Border Management Division – I

o Border Management Division – II

Q73. With reference to the Joint Comprehensive Plan of Action (JCPOA), examine which of the following

statements are correct? 1. It deals with western economic sanctions against Russia.

2. It was signed between P5 countries & the EU.

3. Russia has recently breached the terms of the agreement.

Options:

a. 1 only

b. 2 and 3 only c. 3 only

d. None of the above

Answer: d

Explanation:

The Iran Nuclear Deal, formally known as the Joint Comprehensive Plan of Action (JCPOA) is an agreement on the Iranian nuclear program. Hence Statement 1 is not correct.

It was signed in Vienna on 14 July 2015, between Iran and the P5+1 (the five permanent members of the

United Nations Security Council—China, France, Russia, United Kingdom, United States—plus

Germany). Hence Statement 2 is not correct.

The Iran Nuclear Deal has often been in the news from time to time, primarily focusing on the frosty

relationship between Iran and the United States. Hence Statement 3 is not correct.

Q74. ‘PM SVANidhi’ scheme was launched to benefit -

a. Small and marginal farmers b. Tribal artisans

c. Street vendors

d. Women-led Self-help Groups

Answer: c

Explanation:

The PM Street Vendor’s AtmaNirbhar Nidhi (PM SVANidhi) was launched by the Ministry of Housing and

Urban Affairs on June 01, 2020, for providing affordable Working Capital loans to street vendors to resume their livelihoods that have been adversely affected due to Covid-19 lockdown.

It is a Central Sector Scheme. No funds are released to the States for disbursal to beneficiaries. The loan

amount is directly released to the beneficiaries by the Lending Institutions.

Street Vendors can avail of a working capital loan of up to Rs. 10,000, which is repayable in monthly

instalments in the tenure of one year.

Page 39: BYJU'S UPSC Monthly Magazine March 2022 Answer Key

On timely/early repayment of the loan, an interest subsidy @ 7% per annum will be credited to the bank

accounts of beneficiaries through Direct Benefit Transfer on a quarterly basis.

There will be no penalty on the early repayment of the loan.

Implemented by Small Industries Development Bank of India (SIDBI).

Hence option C is correct.

Q75. If you withdraw 1,00,000 in cash from your Demand Deposit Account at your bank, the immediate effect on

aggregate money supply in the economy will be [UPSC 2020]

a. To reduce it by 1,00,000

b. To increase it by 1,00,000

c. To increase it by more than 1,00,000 d. To leave it unchanged

Answer: d

Explanation:

A demand deposit account is one in which funds are maintained in a bank account and can be withdrawn

at any time.

Demand Deposit Accounts include Current and Savings Accounts.

The entire currency in circulation plus the public’s demand deposits with banks make up the money

supply.

Hence, Money Supply = Currency with public + Currency in the bank.

When you withdraw money from the bank, it is converted into the currency you have on hand, but it has no effect on the money supply’s value. Hence, there will be no change in the aggregate money supply.

Hence option D is correct.

Q76. Which amongst the given States have introduced an ‘anti-lynching Bill’ to deal with mob-lynchings and hate crimes?

1. Karnataka

2. Maharashtra 3. Manipur

4. Rajasthan

Options: a. 1 only

b. 1 and 2 only

c. 2, 3 and 4 only d. 3 and 4 only

Answer: d

Explanation:

Bills have been passed against mob lynching in the past four years by at least 4 States.

They have not been implemented, with the Union government taking a view that lynching is not defined as a

crime under the Indian Penal Code (IPC).

Year Development Description

Page 40: BYJU'S UPSC Monthly Magazine March 2022 Answer Key

2018 Manipur Assembly passed the Manipur Protection from Mob

Violence Bill, 2018

Recommending life imprisonment for those

involved in mob violence if it

led to death. The Bill is still

being examined by the Ministry.

2019 Rajasthan Assembly passed the

Rajasthan Protection from

Lynching Bill, 2019

Providing for life

imprisonment and a fine

ranging from ₹1 lakh to ₹5 lakh to those convicted in cases

of mob lynching leading to the

victim’s death.

Hence Option ‘d’ is correct.

Q77. Which of the following statements is/are correct?

1. ‘Dictionary of Martyrs of India’s Freedom Struggle’ was commissioned by the Ministry of Culture to

commemorate the 150th anniversary of the uprising of 1857. 2. It contains an account of the martyrs from India’s First War of Independence in 1857 to India’s

Independence in 1947.

3. It was compiled by the Indian Council of Historical Research (ICHR).

Options:

a. 1 only

b. 1 and 2 only c. 1 and 3 only

d. 1, 2 and 3

Answer: d

Explanation:

The Ministry of Culture has released the Dictionary of Martyrs of India’s Freedom Struggle in 2019. Hence Statement 1 is correct.

It contains an account of the martyrs from India’s First War of Independence in 1857 to India’s

Independence in 1947. Hence Statement 2 is correct.

The project was compiled by the Indian Council of Historical Research (ICHR) to commemorate the

150th anniversary of the uprising of 1857. Hence Statement 3 is correct.

It includes ex-INA or ex-military personnel who died fighting the British.

It includes the martyrs of the 1857 Uprising, Jallianwala Bagh Massacre (1919), Non-Cooperation

Movement (1920-22), Civil Disobedience Movement (1930-34), Quit India Movement (1942-44), Revolutionary Movements (1915-34), Kisan Movements, Tribal Movements, Agitation for Responsible

Government in the Princely States (Prajamandal), Indian National Army (INA, 1943-45), Royal Indian

Navy Upsurge (RIN, 1946), etc.

Information of about 13,500 martyrs has been recorded in these volumes.

Q78. Foreign funding restrictions on NGOs are placed under which law?

a. Foreign Contribution Regulations Act b. Prevention of Money Laundering Act

c. Reserve Bank of India Act

Page 41: BYJU'S UPSC Monthly Magazine March 2022 Answer Key

d. Conservation of Foreign Exchange and Prevention of Smuggling Activities Act

Answer: a

Explanation:

FCRA was enacted with an aim to regulate the utilization of foreign contributions/hospitality by individuals, associations to keep it consistent with the values of a sovereign, democratic republic.

The FCRA was enacted in 1976 in order to maintain strict control over voluntary organizations and

political associations that received foreign funding.

It is an act passed for regulating and prohibiting the acceptance and utilization of foreign contribution or

foreign hospitality by companies, associations or individuals for such activities that could prove to be detrimental to the national interest and for matters connected therewith or incidental thereto.

Hence Option A is correct.

Q79. Which of the following statements is/are correct?

1. The International Solar Alliance (ISA) was launched by Prime Minister Modi ahead of the 2015 Paris

summit of the Climate Change Convention.

2. Its membership has been limited to only those countries that fall within the tropics. 3. The alliance has partnered with the World Bank to launch the Global Solar Atlas.

Options:

a. 1 only

b. 1 and 2 only c. 1 and 3 only

d. 2 and 3 only

Answer: c

Explanation:

In 2015, PM Narendra Modi and former President of France, Francois Hollande announced the launch of

the International Solar Alliance (ISA) at the 2015 Paris summit. Hence Statement 1 is correct.

The aim of this organization is to increase the use of solar energy. This organization was backed by 120 countries present in the tropical belt.

The International Solar Alliance is a group of nations that lie within the Tropics (Tropic of Cancer and

Tropic of Capricorn) and receive sunshine for more than 300 days. Hence Statement 2 is not correct.

The World Bank, in partnership with the International Solar Alliance (ISA), launched the Global Solar

Atlas. Hence Statement 3 is correct. o It is a free, web-based tool to help investors and policymakers identify potential sites for solar

power generation virtually anywhere in the world.

Q80. In India, the use of carbofuran, methyl parathion, phorate and triazophos is viewed with apprehension. These chemicals are used as – [UPSC 2019]

a. pesticides in agriculture

b. preservatives in processed foods c. fruit-ripening agents

d. moisturizing agents in cosmetics

Answer: a

Page 42: BYJU'S UPSC Monthly Magazine March 2022 Answer Key

Explanation:

The state Agriculture Department in Kerala ordered a ban on the use of several dangerous pesticides in

agriculture in 2019 in order to encourage organic farming.

During this time, the Kerala Agriculture University was urged to come up with alternatives to pesticides

including carbofuran, phorate, methyl parathion, monocrotophos, methyl demethon, prophenophos, and

triazophos, which are all banned. Pesticides including acephate, carbaryl, dimethoate, and flubendiamide, according to Kerala Agriculture University, are less dangerous.

Hence Option A is correct.

Q81. Consider the following statements with respect to International Court of Justice (ICJ): 1. States that ratify the Rome Statute become parties to the ICJ.

2. The ICJ is composed of 15 judges and they have a tenure of six years each.

3. The ICJ can suo moto take up a case in case of genocide.

Which of the statements given above is/are correct?

a. 1 and 2 only

b. 3 only c. 1, 2 and 3

d. None

Answer: d

Explanation:

Statement 1 is not correct. All the 193 member states of the UN are automatically parties to the Court.

Those nations that are not members of the UN may become parties to the Court’s statute with the help of the

Article 93 procedure.

Statement 2 is not correct. The ICJ is composed of 15 judges and they have a tenure of nine years each.

Statement 3 is not correct. The ICJ cannot take up a case suo moto. It can only hear cases or disputes

when requested by States.

Q82. Which of the following statements is/are correct?

1. The power to grant citizenship lies with the Home Ministry. 2. The power of the Government of India to expel foreigners is absolute and unlimited.

Options: a. 1 only

b. 2 only

c. Both

d. None

Answer: c

Explanation:

Statement 1 is correct. The power to grant citizenship lies with the Home Ministry.

Statement 2 is correct. The Foreigners Act confers the power to expel foreigners from India. It vests the

Union Government with absolute and unfettered discretion. Hence, the power of the Government of India

to expel a foreigner is absolute and unlimited.

Page 43: BYJU'S UPSC Monthly Magazine March 2022 Answer Key

Q83. With respect to the Silk Industry, which of the following statements is/are incorrect? 1. India is the largest silk producer in the world.

2. India is the only country producing all the five known commercial silks.

3. Arani silk from Andhra Pradesh has received Geographical Indication (GI) status.

Options:

a. 1 only

b. 2 only c. 1 and 3 only

d. 2 and 3 only

Answer: c

Explanation:

Statement 1 is not correct. China is the world’s largest producer and supplier of silk. India is the second-

largest silk producer in the world.

Statement 2 is correct. India has the unique distinction of being the only country producing all the five known commercial silks, namely, mulberry, tropical tasar, oak tasar, eri and muga.

Statement 3 is not correct. Arani Silk is from Tamil Nadu.

Q84. With respect to the Deep Ocean Mission, which of the following statements is/are correct? 1. It is a central sector scheme where the Government aims to conduct the exploration of the underwater

world on similar lines as ISRO does for space.

2. This ambitious project will be managed by the Ministry of Science and Technology.

Options:

a. 1 only

b. 2 only c. Both

d. None

Answer: a

Explanation:

Statement 1 is correct. It is a central sector scheme where the Government aims to conduct the exploration of the underwater world on similar lines as ISRO does for space.

Statement 2 is not correct. The project will be managed by the Ministry of Earth Sciences.

Q85. Consider the following animals: (UPSC CSE 2021)

1. Hedgehog 2. Marmot

3. Pangolin

To reduce the chance of being captured by predators, which of the above organisms rolls up/roll up and

protects/protect its/their vulnerable parts?

a. 1 and 2 only

b. 2 only c. 3 only

d. 1 and 3 only

Page 44: BYJU'S UPSC Monthly Magazine March 2022 Answer Key

Answer: d

Explanation:

Options 1 and 3 are correct.

Hedgehog – these are small nocturnal spiny mammals and when they are frightened they roll into a ball so

that a predator will feel the full brunt of its sharp spines.

Pangolin – these are the only mammals wholly-covered in scales and they use those scales by rolling up

to protect themselves from predators in the wild.

Marmots are large ground squirrels and are not known to roll up and protect themselves.

Q86. With respect to the Organisation of Islamic Cooperation (OIC), which of the following statements is/are

correct? 1. It is the second-largest organization after the United Nations.

2. Iran was suspended from OIC after the Yemen crisis.

3. The OIC has permanent delegations to the United Nations and the European Union.

Options:

a. 2 only

b. 1 and 3 only c. 1, 2 and 3

d. None

Answer: b

Explanation:

Statement 1 is correct, it is the second-largest intergovernmental body next to the United Nations.

Statement 2 is not correct, Iran has not been suspended from OIC.

Statement 3 is correct, the OIC has permanent delegations to the United Nations and the European Union.

Q87. Consider the following statements with respect to P-Waves and S-Waves:

1. P-Waves create troughs and crests in the material through which they pass, S-waves create density

differences in the material leading to stretching and squeezing of the material. 2. P-Waves can travel through solid, liquid, and gas, S-Waves can only travel through solids and liquids.

Which of the statements given above is/are correct? a. 1 only

b. 2 only

c. Both d. None

Answer: d

Explanation:

Statement 1 is not correct.

o P-waves create density differences in the material leading to stretching (rarefaction) and

squeezing (compression) of the material. o S-waves travel perpendicular to wave direction in a vertical plane, hence they create troughs and

crests in the material through which they pass.

Statement 2 is not correct.

o P waves are the fastest seismic waves and can move through solid, liquid, or gas.

Page 45: BYJU'S UPSC Monthly Magazine March 2022 Answer Key

o S waves are slower and can travel only through solids.

Q88. Which of the following has/have been accorded the Geographical Indication (GI) Tag?

1. Darjeeling Tea

2. Basmati Rice 3. Nagpur Oranges

4. Naga Mircha

5. Jalgaon Banana

Options:

a. 1 and 2 only b. 2, 3, 4 and 5 only

c. 1, 3 and 4 only

d. 1, 2, 3, 4 and 5

Answer: d

Explanation:

All of the above have been accorded Geographical Indication (GI) Tag.

Hence option d is correct.

Q89. Consider the following statements with respect to Sawfish:

1. Sawfish are elasmobranchs. 2. Largetooth Sawfish is listed as Critically Endangered on IUCN.

3. Sawfishes are a protected species in India under Schedule I of the Wildlife Protection Act 1972.

Which of the statements given above is/are correct? a. 1 and 2 only

b. 2 and 3 only

c. 1 and 3 only d. 1, 2 and 3

Answer: d

Explanation:

Statement 1 is correct, Sawfish belong to a group of fishes called elasmobranchs that include sharks and

rays.

o Elasmobranchs refer to members of the subclass Elasmobranchii. o The term elasmobranch is derived from the Greek term elasmos, meaning ‘metal plate’, and

branchia, meaning ‘gills’.

o Elasmobranchs have cartilaginous skeletons and gill openings.

Statement 2 is correct, Largetooth Sawfish is listed as Critically Endangered on IUCN.

Statement 3 is correct, Sawfishes are protected under Schedule I of the Wildlife Protection Act 1972 in India.

Q90. Consider the following statements: (UPSC CSE 2021)

1. Moringa (drumstick tree) is a leguminous evergreen tree. 2. Tamarind tree is endemic to South Asia.

3. In India, most of the tamarind is collected as minor forest produce.

4. India exports tamarind and seeds of moringa.

Page 46: BYJU'S UPSC Monthly Magazine March 2022 Answer Key

5. Seeds of moringa and tamarind can be used in the production of biofuels.

Which of the statements given above are correct?

a. 1, 2, 4 and 5

b. 3, 4 and 5 c. 1, 3 and 4

d. 1, 2, 3 and 5

Answer: b

Explanation:

Statement 1 is not correct, Moringa is a leguminous, small deciduous tree (family Moringaceae) native to tropical Asia but also naturalized in Africa and tropical America.

Statement 2 is not correct, Tamarind is popular in the Indian subcontinent and in Central America and

Mexico and is a common ingredient in the cuisine of those regions. It is indigenous to Africa.

Statement 3 is correct, Tamarind is collected mainly as a minor forest produce in India.

Statement 4 is correct, India exports tamarind and seeds of moringa.

Statement 5 is correct, Fruits, leaves and flowers of moringa are used in the food industry, its seeds are

being used to produce biofuels.

Q91. Consider the following statements with respect to the Qutb Shahi dynasty:

1. Badi Baoli was built by Jamsheed Quli Qutb Shah, of the Qutb Shahi dynasty.

2. The dynasty came to an end when the Mughal Emperor Aurangzeb arrested and jailed Sultan Abul Hasan Qutb Shah for the rest of his life

Which of the statements given above is/are correct? a. 1 only

b. 2 only

c. Both

d. None

Answer: b

Explanation:

The Badi Baoli (step well), was built more than 400 years ago by Sultan Qutb-ul-Mulk, the first ruler of

the Qutb Shahi dynasty. Hence statement 1 is not correct.

The Qutb Shahi dynasty or the Golconda Sultanate was known as the ruling family of the sultanate of

Golkonda.

Situated in South India, the Qutb Shahi dynasty was ruled by the Shia Muslims who belonged to the Turkmen tribe from the Turkmenistan-Armenia region.

This dynasty was founded by Sultan Quli Qutub-ul-Mulk in 1518.

The dynasty ruled for 171 years until the Mughal emperor Aurangzeb’s army conquered Golconda in

1687.

The dynasty came to an end in 1687 during the reign of its seventh Sultan Abul Hasan Qutb Shah, when

the Mughal Emperor Aurangzeb arrested and jailed him for the rest of his life in Daulatabad, absorbed

Golconda into the Mughal empire. Hence statement 2 is correct.

Q92. With respect to Golden Langur, which of the following statements is/are correct?

1. It is endemic to the Western Ghats. 2. Its IUCN status is Critically Endangered.

Page 47: BYJU'S UPSC Monthly Magazine March 2022 Answer Key

3. Golden Langur easily recognised by the colour of their fur, changes according to the seasons.

Options:

a. 1 only

b. 1 and 3 only c. 3 only

d. 1, 2 and 3

Answer: c

Explanation:

Gee’s golden langur (Trachypithecus geei) is an Old World monkey found in a small region of western Assam, India and in the neighbouring foothills of the Black Mountains of Bhutan. Hence statement 1 is

not correct.

It is considered sacred by many Himalayan people. IUCN conservation status: Endangered. Hence

statement 2 is not correct.

Golden langurs are easily recognised by the colour of their fur and are distributed in forested habitats,

fragmented and isolated areas. Hence statement 3 is correct.

Q93. The first concrete step towards modern education in India was laid by

a. Charter Act of 1813 b. Wood’s Despatch

c. Macaulay’s Minute on Education

d. Hunter Education Commission

Answer: a

Explanation:

The Charter Act of 1813 passed by the British Parliament renewed the East India Company’s charter for another 20 years. This is also called the East India Company Act, 1813. This act is important in that it

defined for the first time the constitutional position of British Indian territories.

Hence option A is correct.

Q94. Which of the following is/are correctly matched?

Name of the Pass – Region

1. Zoji La – Great Himalayas

2. Banihal – Pir Panjal

3. Photu La – Zaskar

4. Khardung La – Ladakh Range

Options:

a. 1, 2 and 3 only

b. 2 and 4 only

c. 1, 3 and 4 only

d. 1, 2, 3 and 4

Answer: d

Page 48: BYJU'S UPSC Monthly Magazine March 2022 Answer Key

Explanation:

Zoji La is a high mountain pass in the Great Himalayas in the Indian union territory of Ladakh. Located in the Dras, the pass connects the Kashmir Valley to its west with the Dras and Suru valleys to its northeast

and the Indus valley further east.

Banihal pass is a popular pass in Jammu and Kashmir. It is situated in the Pir- Panjal Range.

Photu La is a mountain pass on the Srinagar-Leh highway in the Zanskar Range of the Himalayas in

India.

Khardung La is the gateway to the Nubra and Shyok Valleys in the Ladakh region of Jammu and Kashmir. It is the highest motorable pass in the world.

Q95. With reference to street-lighting, how do sodium lamps differ from LED lamps?

1. Sodium lamps produce light in 360 degrees but it is not so in the case of LED lamps. 2. As street lights, sodium lamps have longer lifespan than LED lamps.

3. The spectrum of visible light from sodium lamps is almost monochromatic while LED lamps offer

significant colour advantages in street lighting.

Select the correct answer using the code given below. [UPSC 2021]

a. 3 only

b. 2 only c. 1 and 3 only

d. 1, 2 and 3

Answer: c

Explanation:

Sodium lamps emit light in all directions, however, this is not the case with LED bulbs. LED lamps offer a 180-degree bulb angle to save energy and provide targeted lighting across large areas. Statement 1 is

correct.

Sodium lights have a shorter lifespan than LED lamps when used as street lighting. The average lifespan

of high-pressure sodium lights is 24,000 hours. Statement 2 is incorrect.

LED bulbs provide considerable colour enhancements in street lighting. LEDs provide a wider range of

colour options than their High-Pressure Sodium counterparts, resulting in better lighting solutions. Statement 3 is correct.

Q96. Consider the following statements with respect to National Green Tribunal (NGT): 1. The Tribunal is bound by the procedure laid down under the Code of Civil Procedure, 1908 and by

principles of natural justice.

2. It draws inspiration from the India’s constitutional provision of Article 21. 3. NGT is mandated to dispose of all the cases within 12 months.

Which of the statements given above is/are correct?

a. 1 and 2 only b. 2 only

c. 1 and 3 only

d. 1, 2 and 3

Answer: b

Explanation:

Page 49: BYJU'S UPSC Monthly Magazine March 2022 Answer Key

The National Green Tribunal (NGT) is not bound by procedure under the Code of Civil Procedure, 1908

or the Indian Evidence Act, 1872 and is guided by principles of natural justice. However, the Tribunal is vested with the powers of a civil court under the Code of Civil Procedure for discharging its functions.

Hence Statement 1 is not correct.

It was formed by replacing the National Environment Appellate Authority. It also draws inspiration from

Article 21 of the India Constitution which assures to provide a healthy environment to the citizens of

India. Hence Statement 2 is correct.

The Tribunal is mandated to make an endeavor for disposal of applications or appeals finally within 6 months of filing the same. Hence Statement 3 is not correct.

Q97. With respect to the Vijayanagara empire, which of the following statements is/are correct? 1. The single biggest item of import to the Vijayanagar empire was horses.

2. Persian traveler Abdur Razzaq visited Vijayanagara during the rule of Deva Raya I.

Options:

a. 1 only

b. 2 only

c. Both d. None

Answer: a

Explanation:

Vijayanagara, the ‘city of victory’, was established in southern Karnataka by two brothers named

Harihara and Bukka.

The Vijayanagara empire’s single most important import was horses. They imported a significant number of Arabian horses. It is to improve the cavalry horse breed in their respective districts. Hence Statement 1

is correct.

Abdur Razzaq was a scholar and explorer from Persia. As an ambassador of Shah Rukh, the Timurid

dynasty king of Persia, he visited the Vijaynagar Kingdom during the reign of Deva Raya II. Hence

Statement 2 is not correct.

Q98. Arrange the following from North to South:

1. Svalbard 2. Jan Mayen

3. Faroe Islands

Options:

a. 1-2-3

b. 2-3-1

c. 3-1-2 d. 1-3-2

Answer: a

Explanation:

Svalbard is a Norwegian archipelago between mainland Norway and the North Pole. It is one of the

world’s northernmost inhabited areas.

Jan Mayen is a Norwegian volcanic island in the Arctic Ocean, with no permanent population.

Page 50: BYJU'S UPSC Monthly Magazine March 2022 Answer Key

The Faroe Islands is a self-governing archipelago, part of the Kingdom of Denmark. It comprises 18

rocky, volcanic islands between Iceland and Norway in the North Atlantic Ocean, connected by road tunnels, ferries, causeways and bridges.

Hence Option A is correct.

Image source: www.intrepidtravel.com

Q99. Boma technique recently seen in news is related to

a. revolutionary concept of 4D technology

b. remote sensing method that uses light in the form of a pulsed laser to measure ranges

c. luring of animals into an enclosure by chasing them through a funnel-like fencing d. process by which a genetically identical copy of a certain bacteria, plant or animal is produced by asexual

reproduction

Answer: c

Explanation:

The Boma capturing technique involves luring animals into an enclosure by chasing them through a funnel-like fencing. It is popular in Africa.

The funnel tapers into an animal selection-cum-loading chute, supported with grass mats and green net to

make it opaque for animals, which are tehn herded into a large vehicle for transport to another location.

This old technique was earlier utilized to capture wild elephants for training and service.

Hence Option C is correct.

Q100. With reference to Madanapalle of Andhra Pradesh, which one of the following statements is correct? [UPSC 2021]

a. Pingali Venkayya designed the tricolor Indian National Flag here.

b. Pattabhi Sitaramaiah led the Quit India Movement of Andhra region from here. c. Rabindranath Tagore translated the National Anthem from Bengali to English here.

d. Madame Blavatsky and Colonel Olcott set up headquarters of Theosophical Society first here.

Answer: c

Explanation:

Page 51: BYJU'S UPSC Monthly Magazine March 2022 Answer Key

Rabindranath Tagore authored the national anthem ‘Jana Gana Mana’ in 1911.

It was translated into English as ‘Morning Song of India’ and given a tune on February 28, 1919 during

Tagore’s brief stay at Madanapalle.

In 1919, Rabindranath Tagore during his brief stay at Theosophical College, Madanapalle translated his Bengali poem/National Anthem ‘Jana GanaMana’ into English as ‘Morning Song of India’.

Before ‘Jana Gana Mana’ was declared the national anthem on January 24, 1950, two days before India

became a republic, then Prime Minister Jawaharlal Nehru had asked musician Herbert Murrill to give his

opinion on the tune.

Hence Option C is correct.

Q101. Consider the following statements with regards to admissions in Kendriya Vidyalaya (KV):

1. A member of parliament can recommend up to 5 students for admissions into KVs each academic year.

2. A discretionary quota given to the Union Education Minister for admission into KVs was scrapped in 2021.

3. The objective of KVs is to cater to the educational needs of children of transferable Central Government

including Defense and Paramilitary personnel by providing a common programme of education. But admissions are open to civilians as well.

Choose the correct code:

a. 1 & 3 only b. 2 & 3 only

c. 1 & 2 only

d. All of the above

Answer: b

Explanation:

All Members of Parliament (MP) of Lok Sabha and Rajya Sabha have provisions to recommend 10

students for admission in Kendriya Vidyalaya every academic year. Hence statement 1 is not correct.

The Central government scrapped the Union Education Minister’s ‘discretionary quota’ for Kendriya

Vidyalaya (KV) admissions starting from the 2021-2022 academic session. Recommendations made by

the Union ministers for admissions into KVs have also been done away with. Hence statement 2 is correct.

Objective:

o To cater to the educational needs of the children of transferable Central Government employees

including Defence and Para-Military personnel by providing a common programme of education; o To pursue excellence and set pace in the field of school education.

Hence statement 3 is correct.

Q102. Consider the following statements with regards to the ‘2+2’ dialogue held by the Government of India: 1. India holds such talks with Australia, Japan and the USA only.

2. It is a format of dialogue where the defense/foreign ministers or secretaries meet with their counterparts

from another country.

3. The USA was the first country with which India began 2+2 dialogue.

Choose the correct code:

a. 1 & 2 only b. 1 & 3 only

c. 2 & 3 only

d. All of the above

Page 52: BYJU'S UPSC Monthly Magazine March 2022 Answer Key

Answer: c

Explanation:

A ‘two plus two dialogue’ is a term used for the installation of a dialogue mechanism between two

countries’ defence and external affairs ministries.

India has a 2+2 format dialogue mechanism on strategic and security issues with four of its key strategic partners, Russia, Australia, the US and Japan. Hence statement 1 is not correct.

The ‘two plus two dialogue’ is an expression used to indicate that two appointed ministers from each

country, the ministers of defence and external affairs, in this case, will meet up to discuss the two

countries’ strategic and security interests. Hence statement 2 is correct.

India held its first 2+2 dialogue with the US in September 2018. The USA was the first country with which India began 2+2 dialogue. Hence statement 3 is correct.

Q103. Consider the following statements with regards to the Padma awards: 1. The Awards are conferred on the recommendations made by the Padma Awards Committee, which is

constituted by the Prime Minister every year.

2. Government servants including those working with PSUs, except doctors and scientists, are not eligible

for these awards. 3. The total number of awards to be given in a year (excluding posthumous awards and to

NRI/foreigners/OCIs) should not be more than 120.

Choose the correct code:

a. 1 & 2 only

b. 2 & 3 only c. 1 & 3 only

d. All of the above

Answer: d

Explanation:

The Padma Awards are conferred on the recommendations made by the Padma Awards Committee, which is constituted by the Prime Minister every year. The nomination process is open to the public. Even

self-nomination can be made.

All persons without distinction of race, occupation, position or sex are eligible for these awards.

However, Government servants including those working with PSUs, except doctors and scientists, are not eligible for these awards.

The total number of awards to be given in a year (excluding posthumous awards and to

NRI/foreigners/OCIs) should not be more than 120.

Hence all the statements are correct.

Q104. Who amongst the following first proposed the idea of “Gandhi Khaddar Fund”? a. Jawaharlal Nehru

b. Pandit Madan Mohan Malviya

c. Vallabhbhai Patel d. Vinoba Bhave

Answer: b

Page 53: BYJU'S UPSC Monthly Magazine March 2022 Answer Key

Explanation:

Pandit Madan Mohan Malviya went around the country, especially to places where Government repression was stronger, such as Assam and Punjab with Rajendra Prasad.

He went to Assam and there launched a movement against opium, with appreciable success.

Elsewhere he exhorted people to take to the constructive programme of the Congress adopted at Bardoli,

such as the boycott of foreign cloth, removal of untouchability, etc.

He launched the “Gandhi Khaddar Fund” to promote the use of Khaddar.

Hence statement B is correct.

Q105. With reference to Ocean Mean Temperature (OMT), which of the following statements is/are correct?

1. OMT is measured up to a depth of 26°C isotherm which is 129 meters in the southwestern Indian Ocean

during January — March.

2. OMT collected during January — March can be used in assessing whether the amount of rainfall in the monsoon will be less or more than a certain long-term mean.

Select the correct answer using the code given below: [UPSC 2020] a. 1 only

b. 2 only

c. Both 1 and 2

d. Neither 1 nor 2

Answer: b

Explanation:

Sea surface temperature (SST) is routinely used for predicting whether the total amount of rainfall that India

receives during the monsoon season will be less or more than the long-term mean of 887.5 mm. Now, scientists from Pune’s Indian Institute of Tropical Meteorology (IITM) find that Ocean Mean Temperature (OMT) has a

better ability to predict this than the Sea Surface Temperature. Compared with SST which has a 60% success rate

of predicting the Indian summer monsoon, OMT has an 80% success rate. OMT is analysed by measuring the

ocean thermal energy during the period from January to March. Using OMT data, scientists are able to predict with 80% probability the monsoon of any year. Hence, statement 2 is correct. SST is restricted to a few

millimetres of the top ocean layer, whereas OMT which is measured up to a depth of 26 degrees C isotherm, is

more stable and consistent, and the spatial spread is also less. The 26 degrees C isotherm is seen at depths varying from 50–100 metres. During January–March, the mean 26 degrees C isotherm depth in the Southwestern Indian

Ocean is 59 metres. Hence, statement 1 is not correct.

Q106. Consider the following statements with regards to the International Criminal Court:

1. Governed by an international treaty called ‘The Rome Statute’, the ICC is the world’s first permanent

international criminal court.

2. India is one of the founding members of the Rome statute. 3. The ICC does not prosecute those under the age of 18 when a crime was committed.

Choose the incorrect statements: a. 1 only

b. 2 only

c. 1 & 3 only

d. All of the above

Answer: b

Page 54: BYJU'S UPSC Monthly Magazine March 2022 Answer Key

Explanation:

The International Criminal Court (ICC) is an intergovernmental organization and international tribunal headquartered in The Hague. It is governed by an international statute known as the Rome Statute. The

Statute entered into force in July 2002. Hence Statement 1 is correct.

It is the first permanent international criminal court in the world. India is not a member of the ICC. Hence

Statement 2 is not correct.

The ICC’s jurisdiction is limited only to natural persons as provided in Article 25 of the Statute. The

minimum age of the alleged perpetrator should be 18 years at the time of the commission of the crime. Hence Statement 3 is correct.

Q107. Consider the following statements with regards to Biochemical Oxygen Demand: 1. BOD is the amount of dissolved oxygen needed by bacteria in decomposing the organic wastes present in

water.

2. The higher value of BOD indicates better/purer quality of water. 3. Since BOD is limited to biodegradable materials, it is not a reliable method of measuring water pollution.

Choose the correct code:

a. 1 & 2 only b. 1 & 3 only

c. 2 & 3 only

d. All of the above

Answer: b

Explanation:

BOD – Biochemical Oxygen Demand is applied to determine the aerobic destructibility of organic

substances. BOD measures the amount of oxygen consumed by microorganisms for the process of

decomposition of the organic matters in the water bodies. Hence Statement 1 is correct.

It indicates the amount of organic pollution present in an aquatic ecosystem. BOD also measures the

chemical oxidation (COD) of inorganic matter. Hence Statement 2 is not correct.

The amount of oxygen in a litre of water is measured in milligrammes of oxygen per litre of water. The higher the BOD value, the lower the DO content of the water. BOD is not a reliable method of measuring

water pollution because it is limited to biodegradable materials. Hence Statement 3 is correct.

Q108. Which of the following crops are covered under the MSP (Minimum Support Price) mechanism by the

Government of India?

1. Raw Jute 2. Arhar

3. Soybean

4. Sunflower seed

5. Raw cotton

Choose the correct code:

a. All except 2 & 4 b. All except 1 & 4

c. 2, 3 & 4 only

d. All of the above

Answer: d

Page 55: BYJU'S UPSC Monthly Magazine March 2022 Answer Key

Explanation:

MSP is a form of government intervention to insure the farmers against a steep decline in the prices of their goods and to help them prevent losses. The government of India sets the MSP twice a year for 24

commodities. They are as follows:

Crops covered under MSP

Hence option D is correct.

Q109. ‘No taxation without representation’ is a slogan which originated from which of the following events?

a. French Revolution

b. American Civil War

c. Russian Revolution d. American Revolution

Answer: d

Explanation:

Page 56: BYJU'S UPSC Monthly Magazine March 2022 Answer Key

During the American Revolution, the slogan “No Taxation without Representation” was first raised in the

Massachusetts Assembly.

No taxation without representation was a colonial American slogan in the years running up to the American Revolution.

It emphasized the colonists’ position that they were unfairly taxed and lacked proper representation in the

British Parliament as Englishmen.

Hence option D is correct.

Q110. Among the following Tiger Reserves, which one has the largest area under “Critical Tiger Habitat”?

a. Corbett

b. Ranthambore

c. Nagarjunsagar-Srisailam d. Sunderbans

Answer: c

Explanation:

Sundarban has a Critical Tiger Habitat area of 1699.62 sq.

Nagarjunsagar-Srisailam Core/Critical Tiger Habitat: 3721 Sq. Km

Critical ‘tiger’ habitats (CTHs), also known as core areas of tiger reserve, are identified under the Wild

Life Protection Act (WLPA), 1972.

Based on scientific evidence, such areas are required to be kept as safe for the purpose of tiger

conservation, without affecting the rights of the Scheduled Tribes or such other forest dwellers.

The notification of CTH is done by the state government in consultation with the expert committee constituted for the purpose.

Hence option C is correct.

Q111. Lapsus$ recently seen in news is a/an a. Virtual personal assistant

b. Decentralized cryptocurrency

c. Artificial intelligence tool that mimics the workings of a human brain d. Data extortion hacking group

Answer: d

Explanation:

Lapsus$ is a hacker group that has recently targeted companies such as Microsoft, Samsung, Okta and

Nvidia.

South America-based Lapsus$ is known for publicly posting details about their hacks and sharing

screenshots of stolen data on platforms such as Telegram and Twitter.

Q112. Consider the following statements:

1. Municipal corporations are established in the states by the acts of the state legislatures concerned, and in the union territories by the President.

2. The Notified area committee may be wholly elected or wholly nominated by the State government.

Which of the statements given above is/are correct? a. 1 only

b. 2 only

c. Both

Page 57: BYJU'S UPSC Monthly Magazine March 2022 Answer Key

d. None

Answer: d

Explanation:

Statement 1 is not correct, Municipal corporations are established in the states by the acts of the state legislatures concerned, and in the union territories by the acts of the Parliament of India and not the

President.

Statement 2 is not correct, Notified area committee is an entirely nominated body, wherein, all the

members including the chairman are nominated by the state government.

Q113. Which of the following is/are the consequences of President’s Rule?

1. The President can declare that the powers of the state legislature are to be exercised by the Union Cabinet. 2. A law made by the Parliament or President or any other specified authority continues to be operative even

after the President’s Rule.

Options: a. 1 only

b. 2 only

c. Both d. None

Answer: b

Explanation:

Statement 1 is not correct, The President can declare that the powers of the state legislature are to be

exercised by the Parliament and not by the Union Cabinet.

Statement 2 is correct, a law made by the Parliament or President or any other specified authority

continues to be operative even after the President’s Rule. Such a law can be repealed or altered or re-enacted by the state legislature.

Q114. Which of the following statements with respect to Abel Prize is/are correct? 1. It is awarded to mathematicians not over 40 years of age at each International Congress of the

International Mathematical Union.

2. It is awarded every four years.

Options:

a. 1 only

b. 2 only c. Both

d. None

Answer: d

Explanation:

Statement 1 is not correct, the Fields Medal is awarded to the most accomplished mathematicians who are

40 or younger every four years at the Congress of the International and not the Abel Prize.

Statement 2 is not correct, the Abel Prize has been awarded annually since 2003.

Q115. With reference to India, consider the following statements:

Page 58: BYJU'S UPSC Monthly Magazine March 2022 Answer Key

1. When a prisoner makes out a sufficient case, parole cannot be denied to such prisoner because it becomes a matter of his/her right.

2. State Governments have their own Prisoners Release on Parole Rules.

Which of the statements given above is/are correct? [UPSC 2021] a. 1 only

b. 2 only

c. Both 1 and 2 d. Neither 1 nor 2

Answer: b

Explanation:

Statement 1 is not correct, as parole is not a matter of right but a concession that may be provided by the

state prison authorities.

Statement 2 is correct, prison is a state subject and states frame their own prison rules.

Q116. With respect to election to the office of President of India, which of the following statements is/are correct?

1. He/she is elected by members of both the houses of the Parliament and members of the Legislative

Assemblies of the States. 2. The President’s election is held in accordance with the system of Proportional representation by means of

a single transferrable vote and the voting is by Open ballot.

3. Any dispute related to his/her election is taken up by Supreme Court (SC). SC’s decision is final.

Options:

a. 1 only

b. 2 and 3 only c. 3 only

d. 1, 2 and 3

Answer: c

Explanation:

Statement 1 is not correct, The electoral college for President’s elections consists of elected members

from: o Lok Sabha and Rajya Sabha

o Legislative Assemblies of the states (Legislative Councils have no role)

o Legislative Assemblies of the Union Territories of Delhi and Puducherry

Statement 2 is not correct, according to Article 55, the President’s election is held in accordance with the system of Proportional representation by means of a single transferrable vote and the voting is by Secret

ballot.

Statement 3 is correct, according to Article 71, any dispute related to the President’s election is taken up

by Supreme Court (SC). SC’s decision is final.

Q117. Which of the following Dams/Power Projects is/are constructed/proposed to be constructed on River

Chenab?

1. Dul Hasti Hydroelectric Plant 2. Kiru Hydroelectric Power Project

3. Pakal Dul Dam

4. Salal Dam

Page 59: BYJU'S UPSC Monthly Magazine March 2022 Answer Key

5. Uri Dam

Options:

a. 1, 2 and 5 only b. 2, 3, 4 and 5 only

c. 1, 2, 3 and 4 only

d. 1, 2, 3, 4 and 5

Answer: c

Explanation:

Dul Hasti Hydroelectric Plant, Kiru Hydroelectric Power Project, Pakal Dul Dam and Salal Dam are

being constructed on River Chenab.

Uri Dam is a hydroelectric power station on the Jhelum River.

Q118. Which of the following Island Countries are in the Pacific Ocean? 1. Comoros

2. Fiji

3. Kiribati 4. Solomon Islands

5. Tonga

Options:

a. 1, 2, 3 and 4 only

b. 2, 3, 4 and 5 only

c. 1, 3, 4 and 5 only d. 1, 2, 3, 4 and 5

Answer: b

Explanation:

Comoros are a group of islands at the northern end of the Mozambique Channel of the Indian Ocean.

All the other countries are in the Pacific Ocean.

Q119. Consider the following statements with respect to appointment of High Court Judges:

1. The Judges of a high court are appointed by the Governor.

2. A distinguished Jurist in the opinion of the Governor can be appointed as a High Court Judge.

Options:

a. 1 only

b. 2 only c. Both

d. None

Answer: d

Explanation:

Statement 1 is not correct, the Judges of the High Court are appointed by the President and not by the

governor.

Page 60: BYJU'S UPSC Monthly Magazine March 2022 Answer Key

Statement 2 is not correct, a distinguished Jurist in the opinion of the President can be appointed as a

High Court Judge.

Q120. Which amongst the following is regarded as the mother of all gharanas as it is considered to be the oldest? a. Gwalior Gharana

b. Dilli Gharana

c. Agra Gharana d. Patiala Gharana

Answer: a

Explanation:

Gwalior Gharana is the oldest and is also regarded as the mother of all other Gharanas.

Q121. Chamayavilakku ritual is a practice in the state of a. Tamil Nadu

b. Karnataka

c. Kerala d. Andhra Pradesh

Answer: c

Explanation:

Chamayavilakku ritual is a part of the annual festival of the Chavara Kottankulangara Devi temple in

Kollam, Kerala.

Q122. The Chauri-Chaura Incident took place in which amongst the following districts in the United Provinces

(present-day Uttar Pradesh)?

a. Benares b. Gorakhpur

c. Jhansi

d. Rae Bareli

Answer: b

Explanation:

The Chauri Chaura incident took place on 4 February 1922 at Chauri Chaura in the Gorakhpur district of the United Provinces.

Q123. With reference to India, consider the following statements: 1. Retail investors through demat account can invest in ‘Treasury Bills’ and ‘Government of India Debt

Bonds’ in primary market.

2. The ‘Negotiated Dealing System-Order Matching’ is a government securities trading platform of the

Reserve Bank of India. 3. The ‘Central Depository Services Ltd.’ is jointly promoted by the Reserve Bank of India and the Bombay

Stock Exchange.

Which of the statements given above is/are correct?

Page 61: BYJU'S UPSC Monthly Magazine March 2022 Answer Key

a. 1 only b. 1 and 2 only

c. 3 only

d. 2 and 3 only

Answer: b

Explanation:

Statement 1 is correct, the RBI has announced that retail investors will be provided the access to buy Government securities online through a Non-Competitive bidding way, directly from Stock Exchanges.

Statement 2 is correct, the “Negotiated Dealing System Order matching” is a government securities

trading platform of the RBI.

Statement 3 is not correct, CDSL is promoted by Bombay Stock Exchange jointly with banks such as

State Bank of India, Bank of India, Bank of Baroda, HDFC Bank, Standard Chartered Bank and Union Bank of India.

Q124. With respect to Gram Nyayalayas Act, 2008, which of the following statements is/are correct?

1. The Gram Nyayalayas have both civil and criminal jurisdiction 2. The Act does not make the setting up of Gram Nyayalayas mandatory.

3. The District Courts may, by notification, make rules for carrying out the provisions of this Act.

Options:

a. 1 and 2 only

b. 2 and 3 only c. 1 and 3 only

d. 1, 2 and 3

Answer: a

Explanation:

The Gram Nyayalayas Act, 2008 was enacted to provide for the establishment of Gram Nyayalayas at the

grassroots level.

It aims at providing access to justice to the citizens at their doorsteps and to ensure that opportunities for securing justice are not denied to any citizen due to social, economic or other disabilities.

The Gram Nyayalayas have both civil and criminal jurisdiction over the offenses and nature of suits

specified in the First and Second schedules of the Act. Hence Statement 1 is correct.

As per Section 3 (1) of the Gram Nyayalayas Act, 2008, the State Governments are responsible for

establishing Gram Nyayalayas in consultation with the respective High Courts. However, the Act does not make the setting up of Gram Nyayalayas mandatory. Hence Statement 2 is correct.

The High Court(Not district Court) may, by notification, make rules for carrying out the provisions of

Power of High of this Act. Hence Statement 3 is not correct.

Q125. Consider the following statements with respect to Kalanamak rice:

1. Kalanamak rice has been cultivated since the Buddhist period

2. Kalanamak is highly resistant to rice diseases such as panicle blast, stem rot and brown spot 3. Kalanamak Rice has been granted the Geographical Indication (GI) Tag

Which of the statements given above is/are correct?

a. 1 only

Page 62: BYJU'S UPSC Monthly Magazine March 2022 Answer Key

b. 2 and 3 only c. 1 and 3 only

d. 1, 2 and 3

Answer: d

Explanation:

Kalanamak Rice is a non-basmati scented rice variety.

It is grown in the Terai region of Uttar Pradesh and it is named so because of its black husk.

Kalanamak rice has been cultivated since the Buddhist period (600 BC). The kalanamak grains were

found from excavation of Kapilvastu. Hence Statement 1 is correct. o Kapilvastu, the Kingdom of King Śuddhodana, father of Gautama Buddha is located in Terai of

Nepal.

Kalanamak is also highly resistant to notorious, and in India common, rice diseases such as panicle blast,

stem rot and brown spot. Bacterial blight is quite rarely observed. Hence Statement 2 is correct.

In 2021 the government of Uttar Pradesh celebrated Kalanamak Rice Festival in order to promote the One district and One Product campaign.

The Kalanamak rice from the Siddharthnagar district is conferred with the GI tag.Hence Statement 3 is

correct.

Q126. Consider the following statements with respect to Union Public Service Commission (UPSC): 1. The Chairman and members of the commission hold office for a term of five years or until they attain the

age of 65 years.

2. The Chairman of UPSC is eligible for further employment in the Government of India 3. The UPSC has to be consulted while making reservations of appointments in favor of backward class of

Citizens

Which of the statements given above is/are incorrect?

a. 1 only

b. 3 only

c. 1, 2 and 3 d. None

Answer: c

Explanation:

The Union Public Service Commission (UPSC) consists of a chairman and ten members. The terms and

conditions of service of the chairman and members of the Commission are governed by the Union Public Service Commission (Members) Regulations, 1969.

The chairman and members of the Commission hold office for a term of six years or until they attain the

age of 65 years, whichever is earlier. Hence Statement 1 is not correct.

The Chairman of the UPSC shall be ineligible for any further employment either under the Government

of India or under the Government of a State. Hence Statement 2 is not correct.

The UPSC is not consulted on the following matters: While making reservations of appointments or posts

in favor of any backward class of citizens. While taking into consideration the claims of scheduled castes and scheduled tribes in making appointments to services and posts. Hence Statement 3 is not correct.

Q127. Who was the first ruler who tried to take his message to the people through inscriptions? a. Chandragupta Maurya

b. Bindusara

Page 63: BYJU'S UPSC Monthly Magazine March 2022 Answer Key

c. Ashoka d. Dasharatha Maurya

Answer: c

Explanation:

The most famous Mauryan ruler was Ashoka. He was the first ruler who tried to take his message to the

people through inscriptions. Most of Ashoka’s inscriptions were in Prakrit and were written in the Brahmi

script. Kalinga is the ancient name of coastal Orissa.

Hence Option c is correct.

Q128. Consider the following statements in respect of the ICC World Test Championship:

1. The finalists were decided by the number of matches they won. 2. New Zealand was ranked ahead of England because it won more matches than England.

Which of the above statements is/are correct?

a. 1 only b. 2 only

c. Both 1 and 2

d. Neither 1 nor 2

Answer: d

Explanation:

Every Test series was worth 120 points in the 2019-2021 cycles, with those points distributed evenly

across the matches.

All teams played the same number of series, and the two teams with the most points at the end of the

cycle were supposed to advance to the final.

The finalists were chosen based on the percentage of available points they accumulated rather than the

number of matches they won. Hence Statement 1 is incorrect.

New Zealand has a higher ranking than England because of the percentage of points they have collected rather than the match they have won. Hence Statement 2 is incorrect.

Q129. Consider the following statements with regards to the Sansad Ratna award: 1. The award was established in 2010 to honor the top performing MPs.

2. The award was established based on the recommendation given by Dr. APJ Abdul Kalam.

3. The selection committee for the award is headed by the Lok Sabha speaker.

Choose the correct statement/s:

a. 1 only

b. 1 & 2 only c. 2 & 3 only

d. All of the above

Answer: b

Explanation:

Sansad Ratna Award was established in 2010 by the Prime Point Foundation and e-magazine PreSense.

Hence statement 1 is correct.

Page 64: BYJU'S UPSC Monthly Magazine March 2022 Answer Key

It aims to honor “top performing” Members of the Indian Parliament, based on the suggestions given by

Dr A.P.J. Abdul Kalam, Former President of India. Hence statement 2 is correct.

Many eminent citizens, including retired bureaucrats, professionals, social workers and academicians of Chennai are associated with the Awards Committee. T S Krishnamurthy, Former Chief Election

Commissioner of India is the ‘Patron’ of the Awards Committee. Hence statement 3 is not correct.

These are the only awards presented by citizens and the Indian civil society to honour the top performing

Parliamentarians in the government of India.

Q130. Which of the following fundamental rights in India is/are available exclusively to minority communities

only?

a. Article 29 & 30 b. Article 29 only

c. Article 30 only

d. Article 28, 29 & 30

Answer: c

Explanation:

Article 30 states about the Right of minorities to establish and administer educational institutions o All minorities, whether based on religion or language, shall have the right to establish and

administer educational institutions of their choice

o The state shall not, in granting aid to educational institutions, discriminate against any educational institution on the ground that it is under the management of a minority, whether based

on religion or language

Article 30 of the Indian Constitution states the right of minorities to establish and administer educational

institutions.

It says: “All minorities, whether based on religion or language, shall have the right to establish and administer educational institutions of their choice.”

Hence option C is correct.

Q131. Consider the following statements with regards to the Olive Ridley Turtles:

1. They are a part of Schedule I of the Wildlife Protection Act, 1972. 2. They are best known for their unique mass nesting called Arribada.

3. Odisha’s Gahirmatha Marine Sanctuary is known as the world’s largest rookery (colony of breeding

animals) of sea turtles.

Choose the correct code:

a. 1 & 2 only b. 2 & 3 only

c. 1 & 3 only

d. All of the above

Answer: d

Explanation:

Olive Ridley Turtles have been given legal protection under Schedule 1 of the Wildlife Protection Act 1972.

o The Olive Ridley Turtles are also protected by CITES under Appendix 1.

o They are also listed in the Convention on Migratory Species (CMS), also called Bonn

Convention.

Page 65: BYJU'S UPSC Monthly Magazine March 2022 Answer Key

The unique mass nesting of Olive Ridley Turtles (female turtles assemble on the same beach to lay eggs)

is called Arribada.

Gahirmatha Marine Sanctuary is a marine wildlife sanctuary located in Odisha and is a very popular tourist attraction of Odisha in India. It is the world’s largest nesting beach for Olive Ridley Turtles.

Hence all the statements are correct.

Q132. Which of the following sets of nations represent BIMSTEC members?

a. India, Pakistan, Nepal b. Nepal, Bhutan, Myanmar

c. Sri Lanka, Thailand, Cambodia

d. India, Maldives, Thailand

Answer: b

Explanation:

Bay of Bengal Initiative on Multi-Sectoral Technical and Economic Cooperation.

The BIMSTEC states are those which are on the shore or are adjacent to the Bay of Bengal and are

dependent on it.

There are seven nations in BIMSTEC. There are five Southasian nations:

o Bangladesh

o Bhutan o India

o Nepal

o Sri Lanka

There are two southeast Asian nations: o Myanmar

o Thailand

Hence option B is correct.

Q133. With reference to India’s biodiversity, Ceylon frogmouth, Coppersmith barbet, Graychinned minivet and

White-throated redstart are

a. Birds

b. Primates c. Reptiles

d. Amphibians

Answer: a

Explanation:

Ceylon frogmouth, Coppersmith barbet, Graychinned minivet and White-throated redstart are birds.

Srilanka frogmouth are found in the Western Ghats. Its IUCN status is “least concern”. Batrachostomus moniliger is its scientific name.

Coppersmith Barbets, also known as Crimson-breasted Barbets, are a type of bird that can be found

throughout the Indian Subcontinent. It has the least concerned IUCN status.

Psilopogon haemacephalus is its scientific name. From the Himalayas to China, the Grey Chinned

Minivet is a bird species. Its scientific name is Pericrocotus Solaris.

In India, Nepal, Bhutan, and China, the white-throated redstart is a bird species. The IUCN has also

designated it as a species of least concern. Phoenicurus schisticeps is its scientific name.

Hence A is the correct option.

Page 66: BYJU'S UPSC Monthly Magazine March 2022 Answer Key

Q134. Which amongst the following is the best description of the Lasker Award? a. It is awarded annually to honor a living architect or architects

b. It is an award for achievements in newspaper, magazine and online journalism

c. It is a prize awarded annually to grassroots environmental activists

d. It is awarded to persons who have made major contributions to medical science

Answer: d

Explanation:

The Lasker Awards program was created in 1945 by Albert and Mary Lasker to shine a spotlight on

fundamental biological discoveries and clinical advances that improve human health, and to draw

attention to the importance of public support of science.

Lasker Awards are given in the categories of Basic Research, Clinical Research, Special Achievement, and Public Service.

Hence Option D is correct.

Q135. Gidda is a popular folk dance form practiced in the State of

a. Goa b. Himachal Pradesh

c. Odisha

d. Punjab

Answer: d

Explanation:

Gidda, also known as Lokh Naach, is a popular women’s folk dance from Punjab.

It is derived from an ancient ring dance. Gidda is a bright, colorful, and energetic dance performed by

women during social gatherings.

It is accompanied by rhythmic clapping and boliyan. Gidda, like other folk dances, is a very energetic

dance with a lot of leg action.

Hence Option D is correct.

Q136. With respect to the Scheduled Tribes, which of the following statements is/are correct?

1. Presidential order under Article 342 regarding scheduled tribes is final. Court cannot add or subtract any

entry. 2. A person declared as Scheduled Tribe in one state cannot claim benefit of his status for employment,

education or land allotment in any other state on migration.

Options:

a. 1 only

b. 2 only

c. Both d. None

Answer: c

Explanation:

Article 342 deals with the provisions related to Scheduled Tribes.

It states that the President may, with respect to any State or Union territory, and where it is a state, after

consultation with the Governor thereof by public notification, specify the tribes or tribal communities or

Page 67: BYJU'S UPSC Monthly Magazine March 2022 Answer Key

parts of or groups within tribes or tribal communities which shall, for the purposes of this constitution, is deemed to be scheduled tribes in relation to that State or Union Territory, as the case may be.

Presidential order under Article 342 regarding scheduled tribes is final. Court cannot add or subtract any

entry. Hence Statement 1 is correct.

Recently, the Supreme Court said a person declared as Scheduled Caste or Scheduled Tribe in one state

cannot claim benefit of his status for employment, education or land allotment in any other state on

migration. Hence Statement 2 is correct. o For example, being a Scheduled Caste belonging to Punjab and being a permanent resident of

Punjab, cannot claim the benefit of a Scheduled Caste in the state of Rajasthan.

Q137. Consider the following statements:

1. Natural rubber is a polymer of Styrene

2. Rubber Board’s headquarters is located at Kottayam in Kerala. 3. The world’s first Genetically Modified (GM) rubber plant was planted in the state of Assam

Which of the statements given above is/are correct?

a. 1 and 2 only b. 2 and 3 only

c. 1 and 3 only

d. 1, 2 and 3

Answer: b

Explanation:

Natural rubber is a polymer made up of isoprene units with a small percentage of impurities derived from

tree latex. Hence Statement 1 is not correct.

The Rubber Research Institute of India (RRII) in Puthuppally, Kottayam, Kerala, developed the GM

rubber sapling. Hence Statement 2 is correct.

Rubber Board has planted the world’s first genetically modified (GM) rubber plant at its farm in Sarutari,

near Guwahati, in Assam. Hence Statement 3 is correct.

Q138. How is permaculture farming different from conventional chemical farming?

1. Permaculture farming discourages monocultural practices but in conventional chemical farming, monoculture practices are predominant.

2. Conventional chemical farming can cause an increase in soil salinity but the occurrence of such

phenomenon is not observed in permaculture farming. 3. Conventional chemical farming is easily possible in semi-arid regions but permaculture farming is not so

easily possible in such regions.

4. Practice of mulching is very important in permaculture farming but not necessarily so in conventional

chemical farming.

Select the correct answer using the code given below.

a. 1 and 3 b. 1, 2 and 4

c. 4 only

d. 2 and 3

Answer: b

Explanation:

Page 68: BYJU'S UPSC Monthly Magazine March 2022 Answer Key

Permaculture is a nature-inspired, fully integrated design system. Multicropping and integrated farming

systems are encouraged in permaculture farming. A good example is agroforestry. Hence Statement 1 is correct.

Due to the use of crop-specific inputs such as irrigation, chemical fertiliser, and harvesting methods,

chemical farming is better suited to monoculture cropping.

Chemical fertilizers deplete the soil’s fertility over time, resulting in serious issues such as soil

salinization. Permaculture farming, on the other hand, does not have these issues because it uses organic

fertilizers. Hence Statement 2 is correct.

Permaculture is based on the idea that well-designed systems don’t produce waste, and that permaculture

tries to imitate well-designed systems. As a result, permaculture attempts to account for local conditions

such as arid climate. Semi-arid regions are unsuitable for conventional farming.Hence Statement 3 is not

correct.

Mulching is an essential part of Permaculture systems for maximizing efficiency. Mulching is not considered necessary in chemical farming, so most farmers do not use it. Hence Statement 4 is correct.

Q139. Consider the following statements with respect to Cantonment Boards: 1. It is created by the Central Government but administered by the State Government.

2. It consists of partly elected and partly nominated members.

3. The elected and nominated members hold office for a term of five years

Which of the statements given above is/are correct?

a. 1 and 2 only

b. 2 only c. 1 and 3 only

d. 1, 2 and 3

Answer: b

Explanation:

Cantonment Boards are established by the Municipal administration for the civilian population in the

cantonment area.

It works under the administrative control of the Defense Ministry of the Government of India. Thus, it is created as well as administered by the central govt. Hence statement 1 is not correct.

A cantonment board is made up of members who are partly elected and partly appointed. Hence statement

2 is correct.

The elected members serve for a five-year term, while the nominated members (i.e., ex-officio members)

serve for as long as they hold the office in the station. Hence statement 3 is not correct.

Q140. With respect to Alopecia areata, which of the following statements is/are correct?

1. It is a common autoimmune disorder that often results in unpredictable hair loss. 2. This can develop anywhere on the body.

3. The condition can affect anyone regardless of age and gender.

Options: a. 1 only

b. 2 and 3 only

c. 3 only d. 1, 2 and 3

Answer: d

Page 69: BYJU'S UPSC Monthly Magazine March 2022 Answer Key

Explanation:

Alopecia areata is an autoimmune disorder that causes hair loss, often in clumps the size and shape of a

quarter. The amount of hair loss is different for everyone.

Alopecia areata can affect just your scalp or your entire body, and it can be temporary or permanent.

The condition can affect anyone regardless of age and gender, though most cases occur before the age of

30.

Hence all the statements are correct.

Q141. Which of the following differences between hard and soft corals is/are correct?

1. Hard corals secrete calcium-based skeletons, soft corals do not. 2. Hard corals are referred to as reef-building corals whereas soft corals do not contribute to the reef-

building activity.

Options:

a. 1 only

b. 2 only

c. Both d. None

Answer: c

Explanation:

Hard corals, also known as scleractinian and stony corals, produce a rigid skeleton made of calcium

carbonate (CaCO3) in crystal form called aragonite.

Hard corals tend to secrete calcium carbonate underneath their bodies. This turns into a hard, rock-like structure upon which other coral larvae can settle. Soft corals do not secrete calcium-based skeletons.

Hence statement 1 is correct.

Hard corals are the primary reef-building corals. Soft coral, also known as Alcyonacea and ahermatypic

corals, do not produce a rigid calcium carbonate skeleton and do not form reefs. Hence statement 2 is

correct.

Q142. Consider the following statements:

1. The right to strike is a fundamental right. 2. Parliament and the State legislatures have the power to restrict or abrogate the fundamental rights of

members of the armed forces.

Which of the statements given above is/are correct?

a. 1 only

b. 2 only

c. Both d. None

Answer: d

Explanation:

In India, the right to protest is a fundamental right under Article 19 of the Constitution of India. But the

right to strike is not a fundamental right but a legal right and with this right statutory restriction is

attached in the industrial dispute Act, 1947. Hence statement 1 is not correct.

Page 70: BYJU'S UPSC Monthly Magazine March 2022 Answer Key

Parliament (Not State Legislature) may, by law, determine to what extent any of the rights conferred by

this Part shall, in their application to, o the members of the Armed Forces; or

o the members of the Forces charged with the maintenance of public order; or

o persons employed in any bureau or other organization established by the State for purposes of

intelligence or counterintelligence; or o persons employed in, or in connection with, the telecommunication systems set up for the

purposes of any Force, bureau or organization referred to in clauses (a) to (c), be restricted or

abrogated so as to ensure the proper discharge of their duties and the maintenance of discipline among them.

Hence statement 2 is not correct.

Q143. Which one of the following is a reason why astronomical distances are measured in light-years? [UPSC 2021]

a. Distances among stellar bodies do not change

b. Gravity of stellar bodies does not change

c. Light always travels in a straight line d. Speed of light is always the same

Answer: d

Explanation:

A light-year is a unit of measurement for the distance between two points in space.

Because the speed of light is constant throughout the universe and is known to a high degree of precision,

astronomical distances are measured in light-years.

Astronomers can also determine how far back in time they are viewing by measuring in light-years. Everything we see in the night sky has already happened because light takes time to reach our eyes.

Hence option D is correct.

Q144. Consider the following statements with regards to Rhinos in India:

1. Launched in 2005, Indian Rhino Vision 2020 was an ambitious effort to attain a wild population of at least 3,000 greater one-horned rhinos spread over seven protected areas in the Indian state of Assam by the year 2020.

2. Wild-to-wild translocations were an essential part of IRV2020 – moving rhinosfrom densely populated parks

like Manas National Park to ones in need of morerhinos, like Kaziranga National Park. 3. The five rhino range nations (India, Bhutan, Nepal, Indonesia and Malaysia) have signed a declaration ‘The

New Delhi Declaration on Asian Rhinos 2019’ for the conservation and protection of the species.

Choose the correct statement/s:

a. 1 only

b. 1 & 3 only

c. 2 & 3 only d. All of the above

Answer: B Explanation:

Indian Rhino Vision 2020 was established in 2005 with the goal of achieving a wild population of at least

3,000 greater one-horned rhinos spread across seven protected areas in the Indian state of Assam by 2020.

Hence Statement 1 is correct.

Page 71: BYJU'S UPSC Monthly Magazine March 2022 Answer Key

Wild-to-wild translocations were an essential part of IRV2020 – moving rhinos from densely populated

parks like Kaziranga NP, to ones in need of more rhinos, like Manas NP. Hence Statement 2 is not correct.

For the conservation and protection of rhinos, the five rhino range nations (India, Bhutan, Nepal,

Indonesia and Malaysia) signed the 'New Delhi Declaration on Asian Rhinos 2019'. Hence Statement 3 is

correct.

Q145. Consider the following statements with regards to BIMSTEC:

1. Initially, it was formed with four Member States with the acronym ‘BISTEC’ (Bangladesh, India, Sri-Lanka

and Thailand Economic Cooperation). 2. It became renamed ‘BIMST-EC’ in 1997, following the inclusion of Myanmar.

3. With the admission of Nepal and Bhutan in 2004, the name of the grouping was changed to ‘Bay of Bengal

Initiative for Multi-Sectoral Technical and Economic Cooperation’ (BIMSTEC).

Choose the correct code:

a. 1 & 2 only

b. 2 & 3 only c. 1 & 3 only

d. All of the above

Answer: D

Explanation:

The Bay of Bengal Initiative for Multi-Sectoral Technical and Economic Cooperation (BIMSTEC) is a

regional organization comprising seven Member States lying in the littoral and adjacent areas of the Bay of Bengal.

Initially, the economic bloc was formed with four Member States with the acronym ‘BIST-EC’

(Bangladesh, India, Sri Lanka and Thailand Economic Cooperation). Hence Statement 1 is correct.

Following the inclusion of Myanmar in 1997 during a special Ministerial Meeting in Bangkok, the Group

was renamed ‘BIMST-EC’ (Bangladesh, India, Myanmar, Sri Lanka and Thailand Economic

Cooperation). Hence Statement 2 is correct.

With the admission of Nepal and Bhutan at the 6th Ministerial Meeting, the name of the grouping was changed to ‘Bay of Bengal Initiative for Multi-Sectoral Technical and Economic Cooperation’

(BIMSTEC). Hence Statement 3 is correct.

Q146. The MNREGA wages are fixed according to the changes in

a. Consumer Price Index – Agriculture Labor

b. Consumer Price Index – Rural Laborer c. Consumer Price Index – Industrial Workers

d. Consumer Price Index – Rural Urban Combined

Answer: A Explanation:

The MGNREGA wage rates are fixed according to changes in the CPI-AL (Consumer Price Index-

Agriculture Labor), which reflects the increase in the inflation in rural areas.

Hence option A is correct.

Q147. Consider the following statements with regards to Sariska Tiger Reserve:

1. Kankarwadi fort is located in the center of the Reserve and it is said that Mughal emperor Aurangzeb had

imprisoned his brother Dara Shikoh at this fort in struggle for succession to the throne.

2. The Reserve also houses a famous temple of Lord Hanuman at Pandupole related to Pandavas.

Page 72: BYJU'S UPSC Monthly Magazine March 2022 Answer Key

3. Sariska is the first tiger reserve to have successfully relocated Royal Bengal tigers in India.

Choose the correct code:

a. 1 & 2 only

b. 2 & 3 only c. 1 & 3 only

d. All of the above

Answer: D

Explanation:

Kankwari fort was used by Aurangzeb to imprison his brother Dara shikoh in the 17th century AD. This

fort is located in the premises of the Sariska tiger reserve.

Pandupol is an ancient Hanuman temple located inside the Sariska National Park. The Legend is that the Pandavas spent their exile time unknown (Gupta time) here.

Sariska is the first tiger reserve to have successfully relocated Royal Bengal tigers in India and at present

(2020) there are around 20 tigers in the reserve.

Hence All the Statements are correct.

Q148. In India, separation of judiciary from the executive is enjoined by a. the Preamble of the Constitution

b. a Directive Principle of State Policy

c. the Seventh Schedule d. the conventional practice

Answer: B

Explanation:

Article 50 of the Indian Constitution recommends the state to take efforts to separate the judiciary from

the executive in the public services of the State. The Criminal Procedure Code (1973) has affected the

separation of Judiciary from the Executive in pursuance of Article 50 under the Directive Principles of State Policy.

Hence option B is correct

Q149. ISRO’s ‘Project NETRA’, recently seen in the news, is best defined as: a. an early warning system in space to detect debris and other hazards to Indian satellites

b. an early warning system in space to detect any harmful object coming towards the Earth, specifically

India c. a part of India’s indigenous navigation system

d. a program to ensure 24×7 surveillance of India’s border areas

Answer: a

Explanation:

The Network for Space Objects Tracking and Analysis (NETRA) project is an initiative by the Indian Space Research Organisation (ISRO) to develop an early warning system in space to detect debris and

hazards to the Indian satellites and space assets.

Q150. Consider the following statements with regards to Pradhan Mantri Jan Arogya Yojana: 1. Beneficiaries are offered reimbursement of their medical expenditures upto Rs. 5 Lakh per family.

Page 73: BYJU'S UPSC Monthly Magazine March 2022 Answer Key

2. The funding for the scheme is shared – 60:40 for all states and UTs with their own legislature, 90:10 in Northeast states and Jammu and Kashmir, Himachal and Uttarakhand and 100% Central funding for UTs

without legislature.

3. The National Health Authority (NHA) has been constituted as an autonomous entity under the Society

Registration Act, 1860 for the effective implementation of PM-JAY in alliance with state governments.

Choose the correct code:

a. 1 & 2 only b. 2 & 3 only

c. 1 & 3 only

d. All of the above

Answer: b

Explanation:

Statement 1 is not correct- The scheme offers eligible families an insurance cover of Rs. 5 lakh per annum per family.

o However, PMJAY provides cashless and paperless access to services for the beneficiary and

hence reimbursement cannot be availed.

Statement 2 is correct, the funding for the scheme is shared – 60:40 for all states and UTs with their own legislature, 90:10 in Northeast states and Jammu and Kashmir, Himachal and Uttarakhand and 100%

Central funding for UTs without legislature.

Statement 3 is correct, The National Health Authority (NHA) has been constituted as an autonomous

entity under the Society Registration Act, 1860 for effective implementation of PM-JAY in alliance with state governments.

o The State Health Agency (SHA) is the nodal agency responsible for the implementation of AB

PM-JAY in the State.

Q151. Consider the following statements with regards to the International Space Station:

1. The space station flies at an average altitude of 400 miles above Earth.

2. An international partnership of five space agencies from 15 countries operates the International Space Station.

3. It can be seen from Earth without the use of a telescope by night sky observers who know when and

where to look.

Choose the correct code:

a. 1 & 2 only

b. 2 & 3 only c. 1 & 3 only

d. All of the above

Answer: b

Explanation:

Statement 1 is not correct, The space station flies at an average altitude of 400 kilometres above the Earth.

Statement 2 is correct, Five different space agencies representing 15 countries built the International

Space Station and continue to operate it today.

Statement 3 is correct, It can be seen from Earth without the use of a telescope by night sky observers who know when and where to look.

Page 74: BYJU'S UPSC Monthly Magazine March 2022 Answer Key

Q152. The Vital-Vidhvansak, the first monthly journal to have the untouchable people as its target audience was published by

a. Gopal Baba Walangkar

b. Jyotiba Phule

c. Mohandas Karamchand Gandhi d. Bhimrao Ramji Ambedkar

Answer: a

Explanation:

Vital-Vidhvansak (Destroyer of Brahmanical or Ceremonial Pollution) was the first monthly journal to

have the untouchable people as its target audience.

It was published by Gopal Baba Walangkar.